You are on page 1of 100

Taller de Autoaprendizaje No 1a 4/20/19, 4)33 PM

Taller de Autoaprendizaje No 1a
Due: 11:59pm on Sunday, April 21, 2019
You will receive no credit for items you complete after the assignment is due. Grading Policy

Dimensions of Physical Quantities

Learning Goal:
To introduce the idea of physical dimensions and to learn how to find them.

Physical quantities are generally not purely numerical: They have a particular dimension or combination of dimensions
associated with them. Thus, your height is not 74, but rather 74 inches, often expressed as 6 feet 2 inches. Although feet and
inches are different units they have the same dimension--length.

Part A
In classical mechanics there are three base dimensions. Length is one of them. What are the other two?

Hint 1. MKS system

The current system of units is called the International System (abbreviated SI from the French Système
International). In the past this system was called the mks system for its base units: meter, kilogram, and second.
What are the dimensions of these quantities?

ANSWER:

acceleration and mass

acceleration and time

acceleration and charge

mass and time

mass and charge

time and charge

Correct

There are three dimensions used in mechanics: length ( l), mass ( m ), and time ( t). A combination of these three
dimensions suffices to express any physical quantity, because when a new physical quantity is needed (e.g., velocity), it
always obeys an equation that permits it to be expressed in terms of the units used for these three dimensions. One then
derives a unit to measure the new physical quantity from that equation, and often its unit is given a special name. Such new
dimensions are called derived dimensions and the units they are measured in are called derived units.

For example, area A has derived dimensions [A] = l2 . (Note that "dimensions of variable x " is symbolized as [x] .) You can
A= 2
https://session.masteringphysics.com/myct/assignmentPrintView?assignmentID=7287775 Page 1 of 100
Taller de Autoaprendizaje No 1a 4/20/19, 4)33 PM

2
find these dimensions by looking at the formula for the area of a square A = s , where s is the length of a side of the
2 2
square. Clearly [s] = l . Plugging this into the equation gives [A] = [s] = l .

Part B
Find the dimensions [V ] of volume.

Express your answer as powers of length ( l), mass ( m ), and time ( t).

Hint 1. Equation for volume

You have likely learned many formulas for the volume of various shapes in geometry. Any of these equations will
3
give you the dimensions for volume. You can find the dimensions most easily from the volume of a cube V = e ,
where e is the length of the edge of the cube.

ANSWER:

[V ] = l3

Correct

Part C
Find the dimensions [v] of speed.
Express your answer as powers of length ( l), mass ( m ), and time ( t).

Hint 1. Equation for speed


Speed v is defined in terms of distance d and time t as

v= d
t.
Therefore, [v] = [d]/ [t].

Hint 2. Familiar units for speed


You are probably accustomed to hearing speeds in miles per hour (or possibly kilometers per hour). Think about
the dimensions for miles and hours. If you divide the dimensions for miles by the dimensions for hours, you will
have the dimensions for speed.

ANSWER:

[v] = tl

https://session.masteringphysics.com/myct/assignmentPrintView?assignmentID=7287775 Page 2 of 100


Taller de Autoaprendizaje No 1a 4/20/19, 4)33 PM

Correct

The dimensions of a quantity are not changed by addition or subtraction of another quantity with the same dimensions. This
means that ∆v , which comes from subtracting two speeds, has the same dimensions as speed.

It does not make physical sense to add or subtract two quanitites that have different dimensions, like length plus time. You
can add quantities that have different units, like miles per hour and kilometers per hour, as long as you convert both
quantities to the same set of units before you actually compute the sum. You can use this rule to check your answers to any
physics problem you work. If the answer involves the sum or difference of two quantities with different dimensions, then it
must be incorrect.

This rule also ensures that the dimensions of any physical quantity will never involve sums or differences of the base
dimensions. (As in the preceeding example, l + t is not a valid dimension for a physical quantitiy.) A valid dimension will only
2/ 3 2 −2
involve the product or ratio of powers of the base dimensions (e.g. m l t ).

Part D
Find the dimensions [a] of acceleration.
Express your answer as powers of length ( l), mass ( m ), and time ( t).

Hint 1. Equation for acceleration


In physics, acceleration a is defined as the change in velocity in a certain time. This is shown by the equation
a = ∆v/ ∆t. The ∆ is a symbol that means "the change in."

ANSWER:

l
[a] = t2

Correct

± PSS 1.2 Unit Conversions

Learning Goal:
To practice Problem-Solving Strategy 1.2 Unit Conversions.
A gallon of water in the United States weighs about 8.33 lb . In other words, the density of water is 8.33 lb/ g a l . What is the
3 3
density of water in kg / m ? What is the density of water in g / cm ?

Problem-Solving Strategy: Unit conversions

IDENTIFY the relevant concepts:


In most cases, you're best off using the fundamental SI units (meters, kilograms, seconds) within a problem. If you need the

https://session.masteringphysics.com/myct/assignmentPrintView?assignmentID=7287775 Page 3 of 100


Taller de Autoaprendizaje No 1a 4/20/19, 4)33 PM

answer to be in a different set of units, wait until the end of the problem to make the conversion.

SET UP the problem and EXECUTE the solution as follows:


Units are multiplied and divided just like ordinary algebraic symbols. This gives us an easy way to convert a quantity from
one set of units to another. The key idea is to express the same physical quantity in two different units and form an equality.
For example, since 1 min = 60 sec the ratio (1 min)/ (60 sec) equals 1.

EVALUATE your answer:


If you do your units conversion correctly, unwanted units will cancel. Finally, check whether your answer is reasonable. If you
have converted to a smaller unit, for example, the number representing the quantity should be larger.

IDENTIFY the relevant concepts


The physical property of density is given by mass/volume. The SI unit for mass is the kilogram (kg ) and for volume it is the
3 3
cubic meter (m ). Therefore, density should be given in units of kg / m .

SET UP the problem and EXECUTE the solution as follows

Part A
3
Calculate the density of water in kg / m .

Express your answer in kilograms per cubic meter using three significant figures.

Hint 1. Find the conversion factor between pounds and kilograms


Which of the following expressions is the correct conversion factor needed when converting pounds into
kilograms?
ANSWER:

2.20 lb
1.00 kg

1.00 kg
2.20 lb
1.00 lb
2.20 kg

Correct
When you convert pounds into kilograms, you need to multiply by a conversion factor that has pounds
in the denominator and equals 1. For example, you can start from the equality
1.00 kg = 2.20 lb
and derive the conversion factor
1.00 kg
2.20 lb
= 1.

https://session.masteringphysics.com/myct/assignmentPrintView?assignmentID=7287775 Page 4 of 100


Taller de Autoaprendizaje No 1a 4/20/19, 4)33 PM

Hint 2. Find the conversion factor between gallons and cubic meters

Which of the following expressions is the correct conversion factor needed when converting gallons into cubic
meters?
ANSWER:

264 m3
1.00 g a l

264 g a l
1.00 m3

1.00 m3
264 g a l

Correct
When you convert gallons into cubic meters, you need to multiply by a conversion factor that has
gallons in the denominator and equals 1. For example, you can start from the equality
1.00 m 3 = 264 g a l
and derive the conversion factor

1.00 m3
264 g a l
= 1.

ANSWER:

8.33 lb/ g a l = 1000 kg / m 3

Correct
3
If you were to complete the conversion and find the equivalent density in kg / m of 8.33 lb/ g a l , you would
write

lb lb 1.00 kg 264 g a l kg
8.33 gal
= 8.33 × × 1.00 m3
= 1000 m3 .
gal 2.20 lb

Part B
3
Now, convert the density of water to g / cm .

Express your answer in grams per cubic centimeter using three significant figures.

Hint 1. Find the conversion factor between cubic meters and cubic centimeters

Which of the following is the correct relationship between cubic meters and cubic centimeters?

https://session.masteringphysics.com/myct/assignmentPrintView?assignmentID=7287775 Page 5 of 100


Taller de Autoaprendizaje No 1a 4/20/19, 4)33 PM

ANSWER:

1 m 3 = 106 cm 3
1 m 3 = 104 cm 3
1 m 3 = 102 cm 3
1 m 3 = 10−2 cm 3
1 m 3 = 10−4 cm 3
1 m 3 = 10−6 cm 3

ANSWER:

3
1000 kg / m = 1.00 g / cm 3

Correct
3
If you had 1 cm of water (about the size of a sugar cube), it would have a mass of 1 g .

EVALUATE your answer

Part C
The same physical quantity, such as density, can be reported using different units. Above, you found that water has a
3 3
density of 1000 kg / m = 1 g / cm . Because the density of water must be the same regardless of what units you
3
use to measure it, you can conclude that an object whose density is 1 kg / m must be less dense than water. In other
3 3
words, 1 kg / m is less than 1 g / cm .
3 3 3
If you had three different objects with densities of 1 kg / m , 1 g / m , and 1 kg / mm , which object would be the
most dense?
Rank the given densities from most to least dense. To rank items as equivalent, overlap them.

Hint 1. How to approach the problem


If all these densities were given in the same units, you could easily compare the objects to identify the most
dense and least dense. Since the units given are all different, convert all the densities to a common set of units,
3
such as kg / m , before making your comparison.

ANSWER:

https://session.masteringphysics.com/myct/assignmentPrintView?assignmentID=7287775 Page 6 of 100


Taller de Autoaprendizaje No 1a 4/20/19, 4)33 PM

Reset Help

Most dense Least dense

Correct
3
These three densities are readily compared if they are converted to the SI unit of density, kg / m :

1 kg / mm 3 = 109 kg / m 3
1 kg / m 3 = 1 kg / m 3
1 g / m3 = 10−3 kg / m 3

Converting Units: The Magic of 1

Learning Goal:
To learn how to change units of physical quantities.
Quantities with physical dimensions like length or time must be measured with respect to a unit, a standard for quantities with
this dimension. For example, length can be measured in units of meters or feet, time in seconds or years, and velocity in
meters per second.

When solving problems in physics, it is necessary to use a consistent system of units such as the International System
(abbreviated SI, for the French Système International) or the more cumbersome English system. In the SI system, which is
the preferred system in physics, mass is measured in kilograms, time in seconds, and length in meters. The necessity of

https://session.masteringphysics.com/myct/assignmentPrintView?assignmentID=7287775 Page 7 of 100


Taller de Autoaprendizaje No 1a 4/20/19, 4)33 PM

using consistent units in a problem often forces you to convert some units from the given system into the system that you
want to use for the problem.

The key to unit conversion is to multiply (or divide) by a ratio of different units that equals one. This works because
multiplying any quantity by one doesn't change it. To illustrate with length, if you know that 1 inch = 2.54 cm , you can write
2.54 cm
1=
1 inch
.
To convert inches to centimeters, you can multiply the number of inches times this fraction (since it equals one), cancel the
inch unit in the denominator with the inch unit in the given length, and come up with a value for the length in centimeters. To
convert centimeters to inches, you can divide by this ratio and cancel the centimeters.

For all parts, notice that the units are already written after the answer box; don't try to write them in your answer also.

Part A
How many centimeters are there in a length 89.5 inches ?
Express your answer in centimeters to three significant figures.
ANSWER:

227 cm

Correct

Sometimes you will need to change units twice to get the final unit that you want. Suppose that you know how to convert from
centimeters to inches and from inches to feet. By doing both, in order, you can convert from centimeters to feet.

Part B
Suppose that a particular artillery piece has a range R = 3910 ya rds . Find its range in miles. Use the facts that
1 mile = 5280 ft and 3 ft = 1 ya rd.
Express your answer in miles to three significant figures.

Hint 1. Convert yards to feet


The first step in this problem is to convert from yards to feet, because you know how to then convert feet into
miles. Convert 3910 ya rds into feet. Use

3 ft
1= 1 ya rd
.

Express your answer in feet to three significant figures.


ANSWER:

1.17×104 ft

https://session.masteringphysics.com/myct/assignmentPrintView?assignmentID=7287775 Page 8 of 100


Taller de Autoaprendizaje No 1a 4/20/19, 4)33 PM

ANSWER:

3910 ya rds = 2.22 miles

Correct

Often speed is given in miles per hour (mph ), but in physics you will almost always work in SI units. Therefore, you must
convert mph to meters per second (m/ s).

Part C
What is the speed of a car going v = 1.000 mph in SI units? Notice that you will need to change from miles to meters
and from hours to seconds. You can do each conversion separately. Use the facts that 1 mile = 1609 m and
1 hour = 3600 s .
Express your answer in meters per second to four significant figures.

Hint 1. Convert miles to meters

In converting 1.000 mph into meters per second, you will need to multiply by
1609 m
1=
1 mile
.
When you do this, the miles will cancel to leave you with a value in meters per hour. You can then finish the
conversion. What is v = 1.000 mph in meters per hour?
Express your answer in meters per hour to four significant figures.

ANSWER:

v = 1609 m/ hour

Answer Requested

Hint 2. Convert hours to seconds


Which of the following would you multiply 1609 m/ hours by to convert it into meters per second (m/ s)?
ANSWER:

https://session.masteringphysics.com/myct/assignmentPrintView?assignmentID=7287775 Page 9 of 100


Taller de Autoaprendizaje No 1a 4/20/19, 4)33 PM

3600 s
1 hour
3600 s
1 hour
1 hour
3600 s

Correct

ANSWER:

v = 0.4469 m/ s

Correct
Notice that by equating the two values for v, you get 1.000 mph = 0.4469 m/ s . It might be valuable to
remember this, as you may frequently need to convert from miles per hour into more useful SI units. By
remembering this relationship in the future, you can reduce this task to a single conversion.

± A Trip to Europe

Learning Goal:

To understand how to use unit conversion to solve problems.


Unit conversion is not limited to physics problems but is part of our everyday life, correct use of conversion factors is
essential to working through problems of practical importance.
For example, unit conversion could be used in problems involving currency exchange. Say you want to calculate how many
euros you get if you exchange 3600 US dolla rs (USD ), given the exchange rate 1 EUR = 1.20 USD , that is, 1 euro
to 1.20 US dolla rs. Begin by writing down the starting value, 3600 USD . This can also be written as a fraction:

3600 USD
.
1
Next, convert dollars to euros. This conversion involves multiplying by a simple conversion factor derived from the exchange
rate:
1.00 EUR
.
1.20 USD
Note that the "dollar" unit, USD , should appear on the bottom of this conversion factor, since USD appears on the top of
the starting value.

Finally, since dollars are divided by dollars, the units can be canceled and the final result is

( )( ) = 3000 EUR
https://session.masteringphysics.com/myct/assignmentPrintView?assignmentID=7287775 Page 10 of 100
Taller de Autoaprendizaje No 1a 4/20/19, 4)33 PM

( )( 1.00 EUR ) = 3000 EUR .


3600 USD
1 1.20 USD

Currency exchange is only one example of many practical situations where unit conversion may help you to work through
problems. Remember that unit conversion involves multiplying a given value by a conversion factor, resulting in a value in the
new units. The conversion factor can be the ratio of any two quantities, as long as the ratio is equal to one.

You and your friends are organizing a trip to Europe. Your plan is to rent a car and drive through the major European capitals.
By consulting a map you estimate that you will cover a total distance of 5000 km. Consider the euro-dollar exchange rate
given in the introduction and use unit conversion to work through these simple problems.

Part A
You select a rental package that includes a car with an average consumption of 6.00 liters of fuel per 100 km.
Considering that in Europe the average fuel cost is 1.063 euros/ liter , how much (in US dollars) will you spend in fuel
on your trip?
Express your answer numerically in US dollars to three significant figures.

Hint 1. How to approach the problem


Begin with writing the total distance in kilometers. Then multiply this by the consumption rate [in
liters/ (100 km) ] of your rental car to calculate how many liters of fuel are needed. Then compute the total cost
of fuel (in euros) by multiplying your expression by the average fuel cost. Finally, convert the total cost of fuel to
dollars.

Hint 2. Find the unit factor to express the cost of fuel in euros
Which of these unit factors can be used to calculate the cost of fuel in euros?

Hint 1. Conversion factor


When calculating the total cost of fuel in euros, you divide kilometers by kilometers (and cancel out the
kilometer units) and multiply the resulting value in liters by a conversion factor that has the liter unit at the
bottom, so that again you can cancel out units.

ANSWER:

1 EUR
1.063 liters
1.063 EUR
1 liter
1 liter
1.063 EUR
1.063 liters
1 EUR

https://session.masteringphysics.com/myct/assignmentPrintView?assignmentID=7287775 Page 11 of 100


Taller de Autoaprendizaje No 1a 4/20/19, 4)33 PM

Correct

Hint 3. Find the unit factor to convert euros to dollars


Which of these unit factors can be used to convert the total cost of fuel to dollars?

Hint 1. Conversion factor


Since you are converting the cost of fuel from euros to dollars, you need to multiply by a conversion factor
that has the euro unit, EUR, on the bottom, so that you can cancel out units.

ANSWER:

1.20 USD
1 EUR
1.20 EUR
1 USD
1 USD
1.20 EUR
1 EUR
1.20 USD

Correct

Hint 4. Canceling units


You can avoid many mistakes in unit conversion problems if you are careful to cancel units.

ANSWER:

Cost of fuel = 383 USD

Answer Requested

Part B
How many gallons of fuel would the rental car consume per mile?

Express your answer numerically in gallons per mile to three significant figures.

https://session.masteringphysics.com/myct/assignmentPrintView?assignmentID=7287775 Page 12 of 100


Taller de Autoaprendizaje No 1a 4/20/19, 4)33 PM

Hint 1. How to approach the problem


Begin with writing the consumption rate in liters per kilometer. Then multiply this by the unit factor needed to
convert kilometers to miles. Finally convert liters to gallons. Be sure to cancel units so that your answer is
dimensionally consistent.

Hint 2. Find the unit factor to convert kilometers to miles

Which of these unit factors can be used to convert kilometers to miles?

Hint 1. Conversion factor

Since you are converting the car's fuel consumption per kilometer to the car's fuel consumption per mile
you need to multiply by a conversion factor that has the kilometer unit on the top, so that you can cancel
out units.

ANSWER:

1 km
1.609 miles
1.609 km
1 mile
1 mile
1.609 km
1.609 miles
1 km

Correct

Hint 3. Find the unit factor to convert liters to gallons


Which of these unit factors can be used to convert liters to gallons?

Hint 1. Conversion factor


Since you are converting the car's consumption in liters to gallons, you need to multiply by a conversion
factor that has the liter unit at the bottom, so that you can cancel out units.

ANSWER:

https://session.masteringphysics.com/myct/assignmentPrintView?assignmentID=7287775 Page 13 of 100


Taller de Autoaprendizaje No 1a 4/20/19, 4)33 PM

3.785 g a llons
1 liter
1 g a llon
3.785 liters
1 liter
3.785 g a llons

3.785 liters
1 g a llons

Correct

ANSWER:

2.55×10−2 g a llons/ mile

Correct

Part C
What is the average cost, in dollars per gallon, of fuel in Europe?
Express your answer numerically in dollars per gallon to three significant figures.

Hint 1. How to approach the problem


Begin with writing the cost of fuel in euros per liter. Then multiply this by the unit factor needed to convert euros to
dollars. Finally convert liters to gallons. Be sure to cancel units, to make sure that your answer is dimensionally
consistent.

Hint 2. Find the unit factor to convert euros to dollars


Which of these unit factors can be used to convert the price of fuel from euros to dollars?
ANSWER:

https://session.masteringphysics.com/myct/assignmentPrintView?assignmentID=7287775 Page 14 of 100


Taller de Autoaprendizaje No 1a 4/20/19, 4)33 PM

1.20 EUR
1 USD
1 USD
1.20 EUR
1.20 USD
1 EUR
1 EUR
1.20 USD

Correct

Hint 3. Find the unit factor to convert liters to gallons


Which of these unit factors can be used to convert the price of fuel per liter to a price per gallon?
ANSWER:

3.785 g a llons
1 liter
1 g a llon
3.785 liters
1 liter
3.785 g a llons

3.785 liters
1 g a llon

Correct

ANSWER:

4.83 USD/ g a llon

Correct

± Moving at the Speed of Light

https://session.masteringphysics.com/myct/assignmentPrintView?assignmentID=7287775 Page 15 of 100


Taller de Autoaprendizaje No 1a 4/20/19, 4)33 PM

Part A
How many nanoseconds does it take light to travel a distance of 3.30 km in vacuum?
Express your answer numerically in nanoseconds.

Hint 1. How to approach the problem


Light travels at a constant speed; therefore, you can use the formula for the distance traveled in a certain amount
of time by an object moving at constant speed. Before performing any calculations, it is often recommended,
although it is not strictly necessary, to convert all quantities to their fundamental units rather than to multiples of
the fundamental unit.

Hint 2. Find how many seconds it takes light to travel the given distance
8
Given that the speed of light in vacuum is 3.00 × 10 m/ s , how many seconds does it take light to travel a
distance of 3.30 km ?
Express your answer numerically in seconds.

Hint 1. Find the time it takes light to travel a certain distance


How long does it take light to travel a distance r ? Let c be the speed of light.

Hint 1. The speed of an object


The equation that relates the distance s traveled by an object with constant speed v in a time t is

s = vt .

ANSWER:

r⋅c
r
c
c
r

Correct

Hint 2. Convert the given distance to meters

Convert d = 3.30 km to meters.


Express your answer numerically in meters.

Hint 1. Conversion of kilometers to meters


1 km = 103 m
https://session.masteringphysics.com/myct/assignmentPrintView?assignmentID=7287775 Page 16 of 100
Taller de Autoaprendizaje No 1a 4/20/19, 4)33 PM

Recall that 1 km = 103 m .

ANSWER:

3.30 km = 3300 m

Answer Requested

ANSWER:

1.10×10−5 s

Correct
Now convert the time into nanoseconds. Recall that 1 ns = 10−9 s.

ANSWER:

1.10×104 ns

Correct

± Scientific Notation

A number written in scientific notation has the form a × 10 , where 1 ≤ a < 10 and k is an integer, that is, k is one of
k

. . . −3, −2, −1, 0, 1, 2, 3... .

Part A
Determine the values of a and k when 299, 790, 000 is written in scientific notation.
Enter a and k , separated by commas.

ANSWER:

a , k = 2.9979,8

https://session.masteringphysics.com/myct/assignmentPrintView?assignmentID=7287775 Page 17 of 100


Taller de Autoaprendizaje No 1a 4/20/19, 4)33 PM

Answer Requested
8
The value 2.9979 × 10 is the magnitude of the speed of light in meters per second, to five significant figures.
This is really fast: about 675 million miles per hour!

Part B
Determine the values of a and k when 0.51 is written in scientific notation.
Enter a and k , separated by commas.

ANSWER:

a , k = 5.1,-1

Correct
The value 0.51 is the magnitude of the energy associated with a stationary electron, sometimes called its "rest
energy," expressed in millions of electron volts, or MeV.

Part C
3 2
Now consider the expression 4.0 × 10 + 4 × 10 . Determine the values of a and k when the value of this
expression is written in scientific notation.
Enter a and k , separated by commas.

Hint 1. How to approach the problem


Write both numbers such that they have the same power of 10. In this case it would be best to choose the
common power to be either 3 or 2, since these are the two that appear in the expression given. Now you can add
13 13 13
the factors that are multiplied by 10 . For example, 2 × 10 + 3 × 10 = 5 × 10 .
n

ANSWER:

a , k = 4.4,3

Correct
3
The value 4 × 10 = 4000 is the approximate radius of the earth in miles.

Part D
Finally, consider the expression

−11 24
(6.67× )(5.97× )
https://session.masteringphysics.com/myct/assignmentPrintView?assignmentID=7287775 Page 18 of 100
Taller de Autoaprendizaje No 1a 4/20/19, 4)33 PM

−11 24
(6.67×10 )(5.97×10 )
2 .
(6.38×106 )
Determine the values of a and k when the value of this expression is written in scientific notation.
Enter a and k , separated by commas.

Hint 1. A walk-through
Express all the numbers in the expression using scientific notation. To do this, recall that
2
(a × 10k ) = a 2 × 10(2⋅k) . Then you should get a fraction of the form
k k
(a1 ×10 1 )(a2 ×10 2 )
,
a3 ×10k 3
or
a1 ⋅a2 10k 1 ⋅10k 2
a3 k .
10 3

To determine a , you have to divide a 1 ⋅ a 2 by a 3 . This will often give you a number that is not between 1 and 10;
in that case, you have to write it in scientific notation, for instance,
a1 ⋅a2 k4
a3 = a 4 × 10 .

The next step is to find k 5 in the expression


k k2
k
10 5 = 10 1 ⋅10
k .
10 3

Do this by subtracting k 3 from the sum of k 1 and k 2 :

k 5 = (k 1 + k 2 ) − k 3 .
The value of a you are looking for is equal to a 4 ; k is equal to k 4 + k5 .

ANSWER:

a , k = 9.78,0

Correct
This is the magnitude of the acceleration experienced by a freely falling body near the surface of the earth,
2
expressed in meters per second squared (m/ s ). This acceleration is usually denoted by the symbol g. You
2
may learn later that this acceleration is related to the mass Me and radius R e of the earth as g = GMe / R e ,
−11
where G = 6.67 × 10 N ⋅ m 2 / kg 2 is called the gravitational constant, postulated by Sir Isaac Newton
and first measured by Henry Cavendish in 1798.

Significant Figures

https://session.masteringphysics.com/myct/assignmentPrintView?assignmentID=7287775 Page 19 of 100


Taller de Autoaprendizaje No 1a 4/20/19, 4)33 PM

Part A
−27
To seven significant figures, the mass of a proton is 1.672623 × 10 kg . Which of the following choices
demonstrates correct rounding?
Check all that apply.
ANSWER:

1.672 × 10−27 kg

1.67 × 10−27 kg
1.67263 × 10−27 kg

Correct
−27
The number 1.672 × 10 is incorrect because when we round to four significant figures we get 1.673, not
−27
1.672. Similarly, 1.67263 × 10 is incorrect because when we round to six significant figures we get
1.67262, not 1.67263.

Part B

To eight significant figures, Avogadro's constant is 6.0221367 × 10


23
mol −1 . Which of the following choices
demonstrates correct rounding?
Check all that apply.
ANSWER:

6.022 × 1023 mol −1


6.0 × 1023 mol −1
6.02214 × 1023 mol −1

Correct
All these options are correct; they represent different levels of precision, even though the numerical value is
the same.

Made to Order (of Magnitude)

Learning Goal:
To be able to make order-of-magnitude calculations.

https://session.masteringphysics.com/myct/assignmentPrintView?assignmentID=7287775 Page 20 of 100


Taller de Autoaprendizaje No 1a 4/20/19, 4)33 PM

Imagine that a company wants to build a new factory. Such a complex project would involve significant investment in terms of
both time and money. Consequently, before construction can start the company asks for an estimate of the total cost.
Although estimate figures are not exact, they are still helpful: For instance, if the projected cost is three times the amount of
money that the company is willing to spend, the project will be canceled or substantially changed.

Individuals make such estimates all the time. For instance, when you need to drive somewhere for a meeting, you can
roughly predict how much time you will spend on the road and depart accordingly. Of course, the actual travel time is unlikely
to be exactly the same as the estimated one—but it still helps to make an estimate so that you can decide when to leave.

Physicists must frequently make such estimates—known as order-of-magnitude calculations—as part of their job. Depending
on the results of the estimate, a potentially lengthy and costly research project may be postponed, canceled, or redesigned.
Being able to make a quick calculation and get a "ball-park figure" of the expected result is an important skill for a scientist,
involving processes such as identifying relevant information, searching for this information, and using your experience or
background knowledge.

In this problem, you will practice making such order-of-magnitude calculations.

What is the total mass of all the people on earth?


It is impossible, of course, to give an accurate answer to this question. However, it is quite possible to find the order of
magnitude of the answer. All one needs to do is to use some common sense and, possibly, search for relevant reference
information. The calculation can proceed as follows:
9
There are about 7 × 10 people on earth. An average adult male weighs, say, 75 kg ; an average adult female weighs about
60 kg , and an average child will weigh considerably less than 60 kg . Figuring roughly one child per adult, we can reasonably
say that an average person's mass is about 50 kg , which gives the total mass of all humans on our planet as

7 × 109 × 50 = 3.5 × 1011 kg .


Of course, we may be off in our estimates of the average mass or number of people. While it would be unreasonable to say
11
that we know the total mass is 3.5 × 10 kg , we can be reasonably sure that we have the correct order of magnitude; that
is, we have the correct exponent to which the number 10 is raised.

In each of the following problems, you will be asked to make similar estimates.

Part A
How many people can fit into the Pentagon, which was once the largest office building in the world? Assume that
everybody must be standing on the floor.
Round the answer to the nearest power of 10 and then express your answer as the order of magnitude. For
5 5
instance, if your estimated answer is 3 × 10 , enter 5. If your estimated answer is 8.7 × 10 , you should enter
6 (rounding up to the next power of 10).

Hint 1. What reference information should you be looking for?


What information should you be looking for?
Check all that apply.
ANSWER:

https://session.masteringphysics.com/myct/assignmentPrintView?assignmentID=7287775 Page 21 of 100


Taller de Autoaprendizaje No 1a 4/20/19, 4)33 PM

What is the tallest building in the world?

What is the most massive building in the world?

What building in the world has the largest floor area?

In which country is the largest building in the world located?

How many people can be found in the largest building in the world on a typical day?

Hint 2. What numeric quantities do you need to estimate?


What numeric quantities do you need to estimate?
Check all that apply.
ANSWER:

the mass of an average person

the height of an average person

the amount of space the average person needs to work efficiently

the the area that an average person takes up while standing

the volume of an average person

ANSWER:

Correct
Your process for solving this problem might have been something like this:

First, a simple library or Internet search would tell you that the largest building in the world (in terms of the total
floor area) is the Pentagon, the main building of the U.S. Department of Defense. Its total floor area is about
6.5 × 106 ft 2 but only 3.8 × 106 ft 2 can be occupied; it would be hard to stand inside a wall! Assuming
2
that an average person occupies about 2.0 ft when standing (a conservative estimate), we can see that
6
about 1.9 × 10 people (more than three times the entire population of Washington, DC) could fit into the
Pentagon—assuming the floors held up!

Your own answer may have been different from ours or used different details; however, the order of magnitude
was, hopefully, the same.

https://session.masteringphysics.com/myct/assignmentPrintView?assignmentID=7287775 Page 22 of 100


Taller de Autoaprendizaje No 1a 4/20/19, 4)33 PM

Part B
If the entire population of the United States forms a human chain by holding hands, how many times can such a chain be
wrapped around the earth's equator?
Round your answer to the nearest integer.
ANSWER:

Correct
Here is one way to solve this problem:

There are about 300 million people in the United States. The distance between the tips of a person's
outstretched hands is roughly equal to the height of the person. Counting children, we estimate the average
palm-to-palm distance as one meter. Since the equator is about 40 million meters long, division yields about
7.5. However, in this part any answer between 5 and 15 is considered correct—after all, we are just estimating.

Part C
How many times does your heart beat during your lifetime?
Round the answer to the nearest power of 10 and then express your answer as the order of magnitude. For
instance, if your estimated answer is 3 × 105 , enter 5. If your estimated answer is 8.7 × 105 , you should enter
6 (rounding up to the next power of 10).
ANSWER:

Correct
On average, your heart beats about once every second. The number of seconds in the lifetime of an average
U.S. resident is
86, 400 das y × 365 da ys yea rs
yea r × 75 lifetime = 2.4 × 10 s
9

assuming a lifetime of 75 years. Of course, we didn't account for leap years since this is just an estimate.

Part D
Legend has it that, many centuries ago, Archimedes jumped out of his bathtub and ran across town naked screaming
"Eureka!" after he solved an especially difficult problem. Though you may not have thought of things this way before,
when you drink a glass of water, the water that you are drinking contains some water molecules that were in
Archimedes' bathwater that day, because water doesn't get created or destroyed on a large scale. It follows the water
cycle, which includes rain, evaporation, flowing of rivers into the ocean, and so on. In the more than two thousand years
since his discovery, the water molecules from Archimedes' bathwater have been through this cycle enough times that
they are probably about evenly distributed throughout all the water on the earth. When you buy a can of soda, about how

https://session.masteringphysics.com/myct/assignmentPrintView?assignmentID=7287775 Page 23 of 100


Taller de Autoaprendizaje No 1a 4/20/19, 4)33 PM

many molecules from that famous bathtub of Archimedes are there in that can?
Round the answer to the nearest power of 10 and then express your answer as the order of magnitude. For
instance, if your estimated answer is 3 × 105 , enter 5. If your estimated answer is 8.7 × 105 , you should enter
6 (rounding up to the next power of 10).

Hint 1. How to approach the problem


Assume that the water from that bathtub is evenly mixed with the water throughout the surface of the Earth--after
all, it has had more than 2000 years to do so.

Hint 2. Avogadro's number


23
There are about 6.0 × 10 molecules in 18 g of water; in other words, 1 kg of water contains about
3.3 × 1025 molecules .

Hint 3. What to estimate

Estimate the mass of water in Archimedes' bathtub and the total mass of water on earth. (Note that most of the
earth's water is in its oceans.)

ANSWER:

Correct
21
We used the following assumptions: The total mass of water on the earth's surface is 1.4 × 10 kg
(reference information available from many different sources); the mass of the water in the bathtub is guessed
to be 200 kg ; the mass of the water in the can of soda is estimated to be about 0.33 kg ; and 1 kg of water
25
contains about 3.3 × 10 molecules .
25
Thus the total number of molecules in the can is roughly 10 . The fraction of the bathtub molecules in the can
21
is 200/ (1.4 × 10 ) . Therefore, the number of bathtub molecules contained in the can is
25
200×10
21 ≈ 1.4 ×
106 .
1.4×10
Your answer is most likely different but it should still have the same order of magnitude, equal to 6. In case of
some "wilder" assumptions, we count 5 and 7 as correct too.

Running and Walking

Tim and Rick both can run at speed vr and walk at speed vw , with vr > vw . They set off together on a journey of distance
D . Rick walks half of the distance and runs the other half. Tim walks half of the time and runs the other half.

Part A

D
https://session.masteringphysics.com/myct/assignmentPrintView?assignmentID=7287775 Page 24 of 100
Taller de Autoaprendizaje No 1a 4/20/19, 4)33 PM

How long does it take Rick to cover the distance D ?


Express the time taken by Rick in terms of vr , vw , and D .

Hint 1. Compute midpoint for Rick


Find the time that it takes Rick to walk the first half of the distance, that is, to travel a distance D/ 2 at speed vw .
ANSWER:

D
tw,R = 2v w

Hint 2. Compute running time for Rick


Now find the time Rick spends running.
ANSWER:

D
tr,R = 2v
r

Hint 3. What equation to use


Now just add the two times up and you're done.

ANSWER:

D D
tR = 2v + 2v
w r

Correct

Part B
Find Rick's average speed for covering the distance D .
Express Rick's average speed in terms of vr and vw .

Hint 1. Calculate speed using D and time


You were given the total distance and have calculated the total time. Recall that average speed is equal to total
distance traveled divided by the amount of time it took to travel this distance.

ANSWER:

https://session.masteringphysics.com/myct/assignmentPrintView?assignmentID=7287775 Page 25 of 100


Taller de Autoaprendizaje No 1a 4/20/19, 4)33 PM

2v v
va ve,R = v r +r vww

Correct

Part C
How long does it take Tim to cover the distance?
Express the time taken by Tim in terms of vr , vw , and D .

Hint 1. Calculate average speed


Tim walks at speed vw half the time and runs at speed vr for the other half.
Find Tim's average speed in terms of given quantities.

Hint 1. Solve a related problem


Consider a related problem. Tim walks for a time tT / 2 at speed vw and runs for a time tT / 2 at speed
vr .
Find the total distance he travels in time tT in terms of given quantities and tT .
ANSWER:

v w tT v r tT
dT =
2
+ 2

ANSWER:

v w +v r
va ve,T = 2

ANSWER:

tT = v w2D
+v r

Correct

Part D

D
https://session.masteringphysics.com/myct/assignmentPrintView?assignmentID=7287775 Page 26 of 100
Taller de Autoaprendizaje No 1a 4/20/19, 4)33 PM

Who covers the distance D more quickly?


Think logically, but without using the detailed answers in the previous parts.

Hint 1. Consider the relative positions at the midpoint

Imagine that both Rick and Tim do all of their walking before they start to run. Rick will start running when he has
covered half of the total distance. When Tim reaches half of the total distance, will he already have started
running?

ANSWER:

Rick

Tim

Neither. They cover the distance in the same amount of time.

Correct

Part E
In terms of given quantities, by what amount of time, ∆t , does Tim beat Rick?
It will help you check your answer if you simplify it algebraically and check the special case vr = vw .
Express the difference in time, ∆t in terms of vr , vw , and D .
ANSWER:

2
D(v w −v r )
∆t =
2v r v w (v r +v w )

Correct

Part F
In the special case that vr = vw , what would be Tim's margin of victory ∆t(vr = vw ) ?

Hint 1. Think it through


If vr = vw , is the any difference between what Tim and Rick do?

ANSWER:

https://session.masteringphysics.com/myct/assignmentPrintView?assignmentID=7287775 Page 27 of 100


Taller de Autoaprendizaje No 1a 4/20/19, 4)33 PM

∆t(vr = vw ) = 0

Correct

± Average Velocity from a Position vs. Time Graph

Learning Goal:
To learn to read a graph of position versus time and to calculate average velocity.

In this problem you will determine the average velocity of a moving


object from the graph of its position x(t) as a function of time t. A
traveling object might move at different speeds and in different
directions during an interval of time, but if we ask at what constant
velocity the object would have to travel to achieve the same
displacement over the given time interval, that is what we call the
object's average velocity. We will use the notation va ve [t1 , t2 ] to
indicate average velocity over the time interval from t1 to t2 . For
instance, va ve [1, 3] is the average velocity over the time interval
from t = 1 to t = 3.

Part A
Consulting the graph shown in the figure, find the object's average velocity over the time interval from 0 to 1 second.
Answer to the nearest integer.

Hint 1. Definition of average velocity


Average velocity is defined as the constant velocity at which an object would have to travel to achieve a given
displacement (difference between final and initial positions, which can be negative) over a given time interval,
from the initial time ti to the final time tf . The average velocity is therefore equal to the displacement divided by
the given time interval. In symbolic form, average velocity is given by
x(tf )−x(ti )
v a ve [ti , tf ] = tf −ti .

ANSWER:

https://session.masteringphysics.com/myct/assignmentPrintView?assignmentID=7287775 Page 28 of 100


Taller de Autoaprendizaje No 1a 4/20/19, 4)33 PM

va ve [0, 1] = 0 m/ s

Correct

Part B
Find the average velocity over the time interval from 1 to 3 seconds.
Express your answer in meters per second to the nearest integer.

Hint 1. Find the change in position


The final and initial positions can be read off the y axis of the graph. What is the displacement during the time
interval from 1 to 3 seconds?
Express your answer numerically, in meters
ANSWER:

x f − x i = 40 m

Hint 2. Definition of average velocity


Average velocity is defined as the constant velocity at which an object would have to travel to achieve a given
displacement (difference between final and initial positions, which can be negative) over a given time interval,
from the initial time ti to the final time tf . The average velocity is therefore equal to the displacement divided by
the given time interval. In symbolic form, average velocity is given by

x(tf )−x(ti )
v a ve [ti , tf ] = tf −ti .

ANSWER:

va ve [1, 3] = 20 m/ s

Correct
A note about instantaneous velocity. The instantaneous velocity at a certain moment in time is represented by
the slope of the graph at that moment. For straight-line graphs, the (instantaneous) velocity remains constant
over the interval, so the instantaneous velocity at any time during an interval is the same as the average
velocity over that interval. For instance, in this case, the instantaneous velocity at any time from 1 to 3 seconds
is the same as the average velocity of 20 m/ s.

Part C

a ve [0, 3]
https://session.masteringphysics.com/myct/assignmentPrintView?assignmentID=7287775 Page 29 of 100
Taller de Autoaprendizaje No 1a 4/20/19, 4)33 PM

Now find va ve [0, 3] .

Give your answer to three significant figures.

Hint 1. A note on the displacement


Since the object's position remains constant from time 0 to time 1, the object's displacement from 0 to 3 is the
same as in Part B. However, the time interval has changed.

ANSWER:

va ve [0, 3] = 13.3 m/ s

Correct
Note that va ve [0, 3] is not equal to the simple arithmetic average of va ve [0, 1] and va ve [1, 3] , i.e.,
v a ve [0,1]+v a ve [1,3]
, because they are averages for time intervals of different lengths.
2

Part D
Find the average velocity over the time interval from 3 to 6 seconds.
Express your answer to three significant figures.

Hint 1. Determine the displacement


What is the displacement?
Answer to the nearest integer.
ANSWER:

x(6.0) − x(3.0) = -40 m

Correct

Hint 2. Determine the time interval


What is the time interval?
Answer to two significant figures.

ANSWER:

tf − ti = 3.0 s

https://session.masteringphysics.com/myct/assignmentPrintView?assignmentID=7287775 Page 30 of 100


Taller de Autoaprendizaje No 1a 4/20/19, 4)33 PM

Correct

ANSWER:

va ve [3.0, 6.0] = -13.3 m/ s

Correct

Part E
Finally, find the average velocity over the whole time interval shown in the graph.
Express your answer to three significant figures.

Hint 1. Determine the displacement


What is the displacement?

Answer to the nearest integer.


ANSWER:

x(6.0) − x(0.0) = 0 m

ANSWER:

va ve [0.0, 6.0] = 0 m/ s

Correct
Note that though the average velocity is zero for this time interval, the instantaneous velocity (i.e., the slope of
the graph) has several different values (positive, negative, zero) during this time interval.

Note as well that since average velocity over a time interval is defined as the change in position (displacement)
in the given interval divided by the time, the object can travel a great distance (here 80 meters) and still have
zero average velocity, since it ended up exactly where it started. Therefore, zero average velocity does not
necessarily mean that the object was standing still the entire time!

What x vs. t Graphs Can Tell You

To describe the motion of a particle along a straight line, it is often convenient to draw a graph representing the position of the

t
https://session.masteringphysics.com/myct/assignmentPrintView?assignmentID=7287775 Page 31 of 100
Taller de Autoaprendizaje No 1a 4/20/19, 4)33 PM

particle at different times. This type of graph is usually referred to as an x vs. t graph. To draw such a graph, choose an axis
system in which time t is plotted on the horizontal axis and position x on the vertical axis. Then, indicate the values of x at
various times t. Mathematically, this corresponds to plotting the variable x as a function of t. An example of a graph of
position as a function of time for a particle traveling along a straight line is shown below. Note that an x vs. t graph like this
does not represent the path of the particle in space.

Now let's study the graph shown in the figure in more detail. Refer
to this graph to answer Parts A, B, and C.

Part A
What is the overall displacement ∆x of the particle?
Express your answer in meters.

Hint 1. Definition of displacement


The displacement ∆x of the particle is given by the difference between the initial position x 0 at t = 0.0 s and
the position x at t = 50.0 s . In symbols,

∆x = x − x 0 .

Hint 2. How to read an x vs. t graph


Remember that in an x vs. t graph, time t is plotted on the horizontal axis and position x on the vertical axis. For
example, in the plot shown in the figure, x = 16.0 m at t = 10.0 s .

ANSWER:

∆x = 30 m

Correct
In this example, the magnitude of the displacement is also equal to the total distance traveled by the particle (30
m).

https://session.masteringphysics.com/myct/assignmentPrintView?assignmentID=7287775 Page 32 of 100


Taller de Autoaprendizaje No 1a 4/20/19, 4)33 PM

Part B
What is the average velocity va v of the particle over the time interval ∆t = 50.0 s ?
Express your answer in meters per second.

Hint 1. Definition and graphical interpretation of average velocity

The average velocity va v of a particle that undergoes a displacement ∆x along a straight line in a time interval
∆t is defined as
∆x
va v = ∆t
.

In an x vs. t graph, then, the average velocity equals the slope of the line connecting the initial and final
positions.

Hint 2. Slope of a line

The slope m of a line from point A, with coordinates (tA , x A ), to point B, with coordinates (tB , x B ), is equal to
the "rise" over the "run," or
x B −x A
m= tB −tA .

ANSWER:

va v = 0.600 m/ s

Correct
The average velocity of a particle between two positions is equal to the slope of the line connecting the two
corresponding points in an x vs. t graph.

Part C
What is the instantaneous velocity v of the particle at t = 10.0 s ?
Express your answer in meters per second.

Hint 1. Graphical interpretation of instantaneous velocity


The velocity of a particle at any given instant of time or at any point in its path is called instantaneous velocity. In
an x vs. t graph of the particle's motion, you can determine the instantaneous velocity of the particle at any point
in the curve. The instantaneous velocity at any point is equal to the slope of the line tangent to the curve at that
point.

ANSWER:

https://session.masteringphysics.com/myct/assignmentPrintView?assignmentID=7287775 Page 33 of 100


Taller de Autoaprendizaje No 1a 4/20/19, 4)33 PM

v = 0.600 m/ s

Correct
The instantaneous velocity of a particle at any point on its x vs. t graph is the slope of the line tangent to the
curve at that point. Since in the case at hand the curve is a straight line, the tangent line is the curve itself.
Physically, this means that the instantaneous velocity of the particle is constant over the entire time interval of
motion. This is true for any motion where distance increases linearly with time.

Another common graphical representation of motion along a straight line is the v vs. t graph, that is, the graph of
(instantaneous) velocity as a function of time. In this graph, time t is plotted on the horizontal axis and velocity v on the
vertical axis. Note that by definition, velocity and acceleration are vector quantities. In straight-line motion, however, these
vectors have only one nonzero component in the direction of motion. Thus, in this problem, we will call v the velocity and a
the acceleration, even though they are really the components of the velocity and acceleration vectors in the direction of
motion.

Part D
Which of the graphs shown is the correct v vs. t plot for the motion described in the previous parts?

Hint 1. How to approach the problem

Recall your results found in the previous parts, namely the fact that the instantaneous velocity of the particle is

https://session.masteringphysics.com/myct/assignmentPrintView?assignmentID=7287775 Page 34 of 100


Taller de Autoaprendizaje No 1a 4/20/19, 4)33 PM

constant. Which graph represents a variable that always has the same constant value at any time?

ANSWER:

Graph A

Graph B

Graph C

Graph D

Correct
Whenever a particle moves with constant nonzero velocity, its x vs. t graph is a straight line with a nonzero
slope, and its v vs. t curve is a horizontal line.

Part E
Shown in the figure is the v vs. t curve selected in the previous part. What is the area A of the shaded region under the
curve?
Express your answer in meters.

Hint 1. How to approach the problem


The shaded region under the v vs. t curve is a rectangle whose horizontal and vertical sides lie on the t axis and
the v axis, respectively. Since the area of a rectangle is the product of its sides, in this case the area of the
shaded region is the product of a certain quantity expressed in seconds and another quantity expressed in
meters per second. The area itself, then, will be in meters.

ANSWER:

https://session.masteringphysics.com/myct/assignmentPrintView?assignmentID=7287775 Page 35 of 100


Taller de Autoaprendizaje No 1a 4/20/19, 4)33 PM

A = 30 m

Correct
Compare this result with what you found in Part A. As you can see, the area of the region under the v vs. t
curve equals the overall displacement of the particle. This is true for any velocity curve and any time interval:
The area of the region that extends over a time interval ∆t under the v vs. t curve is always equal to the
displacement over ∆t .

Given Positions, Find Velocity and Acceleration

Learning Goal:
To understand how to graph position, velocity, and acceleration of an object starting with a table of positions vs. time.
The table shows the x coordinate of a moving object. The position is tabulated at 1-s intervals. The x coordinate is indicated
below each time. You should make the simplification that the acceleration of the object is bounded and contains no spikes.

time (s) 0 1 2 3 4 5 6 7 8 9
x (m) 0 1 4 9 16 24 32 40 46 48

Part A
Which graph in best represents the function x(t) , describing
the object's position vs. time?

Hint 1. Meaning of a bounded and nonspiky acceleration


A bounded and nonspiky acceleration results in a smooth graph of x vs. t.

https://session.masteringphysics.com/myct/assignmentPrintView?assignmentID=7287775 Page 36 of 100


Taller de Autoaprendizaje No 1a 4/20/19, 4)33 PM

ANSWER:

Graph 1

Graph 2

Graph 3

Graph 4

Correct

Part B
Which of the following graphs in best represents the function
v(t), describing the object's velocity as a function of time?

Hint 1. Find the velocity toward the end of the motion


Velocity is the time derivative of displacement. Given this, the velocity toward the end of the motion is
__________.
ANSWER:

positive and increasing

positive and decreasing

negative and increasing

negative and decreasing

https://session.masteringphysics.com/myct/assignmentPrintView?assignmentID=7287775 Page 37 of 100


Taller de Autoaprendizaje No 1a 4/20/19, 4)33 PM

Hint 2. What are the implications of zero velocity?


Two of the possible velocity vs. time graphs indicate zero velocity between t = 4 and t = 7 s. What would the
corresponding position vs. time graph look like in this region?
ANSWER:

a horizontal line

straight but sloping up to the right

straight but sloping down to the right

curved upward

curved downward

Hint 3. Specify the characteristics of the velocity function

The problem states that "the acceleration of the object is bounded and contains no spikes." This means that the
velocity ___________.
ANSWER:

has spikes

has no discontinuities

has no abrupt changes of slope

is constant

ANSWER:

Graph 1

Graph 2

Graph 3

Graph 4

https://session.masteringphysics.com/myct/assignmentPrintView?assignmentID=7287775 Page 38 of 100


Taller de Autoaprendizaje No 1a 4/20/19, 4)33 PM

Correct
In principle, you could also just compute and plot the average velocity. The expression for the average velocity
is
x(t )−x(t )
v a vg [t1 , t2 ] = 2t2 −t1 1 .
The notation va vg [t1 , t2 ] emphasizes that this is not an instantaneous velocity, but rather an average over an
interval. After you compute this, you must put a single point on the graph of velocity vs. time. The most
accurate place to plot the average velocity is at the middle of the time interval over which the average was
computed.

Also, you could work back and find the position from the velocity graph. The position of an object is the integral
of its velocity. That is, the area under the graph of velocity vs. time from t = 0 up to time t must equal the
position of the object at time t. Check that the correct velocity vs. time graph gives you the correct position
according to this method.

Part C
Which of the following graphs in best represents the function
a(t), describing the acceleration of this object?

Hint 1. Find the acceleration toward the end of the motion


Acceleration is the time derivative of velocity. Toward the end of the motion the acceleration is __________.

ANSWER:

zero

positive

negative

https://session.masteringphysics.com/myct/assignmentPrintView?assignmentID=7287775 Page 39 of 100


Taller de Autoaprendizaje No 1a 4/20/19, 4)33 PM

Hint 2. Calculate the acceleration in the region of constant velocity


What is the acceleration a over the interval during which the object travels at constant speed?
Answer numerically in meters per second squared.

ANSWER:

a = 0 m/ s 2

Hint 3. Find the initial acceleration


Acceleration is the time derivative of velocity. Initially the acceleration is _________.

ANSWER:

zero

positive

negative

ANSWER:

Graph 1

Graph 2

Graph 3

Graph 4

Correct
In one dimension, a linear increase or decrease in the velocity of an object over a given time interval implies
constant acceleration over that particular time interval. You can find the magnitude of the acceleration using
the formula for average acceleration over a time interval:
v(t )−v(t )
a a vg [t1 , t2 ] = 2t2 −t1 1 .
When the acceleration is constant over an extended interval, you can choose any value of t1 and t2 within the
interval to compute the average.

Kinematic Vocabulary

https://session.masteringphysics.com/myct/assignmentPrintView?assignmentID=7287775 Page 40 of 100


Taller de Autoaprendizaje No 1a 4/20/19, 4)33 PM

One of the difficulties in studying mechanics is that many common words are used with highly specific technical meanings,
among them velocity, acceleration, position, speed, and displacement. The series of questions in this problem is designed to
get you to try to think of these quantities like a physicist.

Answer the questions in this problem using words from the following list:

A. position
B. direction
C. displacement
D. coordinates
E. velocity
F. acceleration
G. distance
H. magnitude
I. vector
J. scalar
K. components

Part A
Velocity differs from speed in that velocity indicates a particle's __________ of motion.
Enter the letter from the list given in the problem introduction that best completes the sentence.

ANSWER:

Correct

Part B
Unlike speed, velocity is a __________ quantity.
Enter the letter from the list given in the problem introduction that best completes the sentence.
ANSWER:

Correct

Part C

https://session.masteringphysics.com/myct/assignmentPrintView?assignmentID=7287775 Page 41 of 100


Taller de Autoaprendizaje No 1a 4/20/19, 4)33 PM

A vector has, by definition, both __________ and direction.


Enter the letter from the list given in the problem introduction that best completes the sentence.
ANSWER:

Correct

Part D
Once you have selected a coordinate system, you can express a two-dimensional vector using a pair of quantities
known collectively as __________.

Enter the letter from the list given in the problem introduction that best completes the sentence.
ANSWER:

Correct

Part E
Speed differs from velocity in the same way that __________ differs from displacement.

Enter the letter from the list given in the problem introduction that best completes the sentence.

Hint 1. Definition of displacement

Displacement is the vector that indicates the difference of two positions (e.g., the final position from the initial
position). Being a vector, it is independent of the coordinate system used to describe it (although its vector
components depend on the coordinate system).

ANSWER:

Correct

https://session.masteringphysics.com/myct/assignmentPrintView?assignmentID=7287775 Page 42 of 100


Taller de Autoaprendizaje No 1a 4/20/19, 4)33 PM

Part F
Consider a physical situation in which a particle moves from point A to point B. This process is described from two
coordinate systems that are identical except that they have different origins.

The __________ of the particle at point A differ(s) as expressed in one coordinate system compared to the other, but the
__________ from A to B is/are the same as expressed in both coordinate systems.
Type the letters from the list given in the problem introduction that best complete the sentence. Separate the
letters with commas. There is more than one correct answer, but you should only enter one pair of comma-
separated letters. For example, if the words "vector" and "scalar" fit best in the blanks, enter I,J.
ANSWER:

A,C

Correct
The coordinates of a point will depend on the coordinate system that is chosen, but there are several other
quantities that are independent of the choice of origin for a coordinate system: in particular, distance,
displacement, direction, and velocity. In working physics problems, unless you are interested in the position of
an object or event relative to a specific origin, you can usually choose the coordinate system origin to be
wherever is most convenient or intuitive.

Note that the vector indicating a displacement from A to B is usually represented as r BA = r B⃗ − r A⃗ .

Part G
Identify the following physical quantities as scalars or vectors.
ANSWER:

https://session.masteringphysics.com/myct/assignmentPrintView?assignmentID=7287775 Page 43 of 100


Taller de Autoaprendizaje No 1a 4/20/19, 4)33 PM

Reset Help

Scalar quantity Vector quantity


velocity average velocity

distance speed position displacement

acceleration

Correct

What Velocity vs. Time Graphs Can Tell You

A common graphical representation of motion along a straight line is the v vs. t graph, that is, the graph of (instantaneous)
velocity as a function of time. In this graph, time t is plotted on the horizontal axis and velocity v on the vertical axis. Note
that by definition, velocity and acceleration are vector quantities. In straight-line motion, however, these vectors have only a
single nonzero component in the direction of motion. Thus, in this problem, we will call v the velocity and a the acceleration,
even though they are really the components of the velocity and acceleration vectors in the direction of motion, respectively.

Here is a plot of velocity versus time for a particle that travels along a straight line with a varying velocity. Refer to this plot to
answer the following questions.

https://session.masteringphysics.com/myct/assignmentPrintView?assignmentID=7287775 Page 44 of 100


Taller de Autoaprendizaje No 1a 4/20/19, 4)33 PM

Part A
What is the initial velocity of the particle, v0 ?

Express your answer in meters per second.

Hint 1. Initial velocity

The initial velocity is the velocity at t = 0 s.

Hint 2. How to read a v vs. t graph

Recall that in a graph of velocity versus time, time is plotted on the horizontal axis and velocity on the vertical
axis. For example, in the plot shown in the figure, v = 2.00 m/ s at t = 30.0 s .

ANSWER:

v0 = 0.5 m/ s

Correct

Part B
What is the total distance ∆x traveled by the particle?

Express your answer in meters.

Hint 1. How to approach the problem


Recall that the area of the region that extends over a time interval ∆t under the v vs. t curve is always equal to
the distance traveled in ∆t . Thus, to calculate the total distance, you need to find the area of the entire region

https://session.masteringphysics.com/myct/assignmentPrintView?assignmentID=7287775 Page 45 of 100


Taller de Autoaprendizaje No 1a 4/20/19, 4)33 PM

under the v vs. t curve. In the case at hand, the entire region under the v vs. t curve is not an elementary
geometrical figure, but rather a combination of triangles and rectangles.

Hint 2. Find the distance traveled in the first 20.0 seconds


What is the distance ∆x 1 traveled in the first 20 seconds of motion, between t = 0.0 s and t = 20.0 s ?
Express your answer in meters.

Hint 1. Area of the region under the v vs. t curve


The region under the v vs. t curve between t = 0.0 s and t = 20.0 s can be divided into a rectangle of
dimensions 20.0 s by 0.50 m/ s , and a triangle of base 20.0 s and height 1.50 m/ s , as shown in the
figure.

ANSWER:

∆x 1 = 25 m

Hint 3. Find the distance traveled in the second 20.0 seconds

What is the distance ∆x 2 traveled in the second 20 seconds of motion, from t = 20.0 s to t = 40.0 s ?
Express your answer in meters.

Hint 1. Area of the region under the v vs. t curve

The region under the v vs. t curve between t = 20.0 s and t = 40.0 s is a rectangle of dimensions
20.0 s by 2.00 m/ s , as shown in the figure.

https://session.masteringphysics.com/myct/assignmentPrintView?assignmentID=7287775 Page 46 of 100


Taller de Autoaprendizaje No 1a 4/20/19, 4)33 PM

ANSWER:

∆x 2 = 40 m

Hint 4. Find the distance traveled in the last 10.0 seconds


What is the distance ∆x 3 traveled in the last 10 seconds of motion, from t = 40.0 s to t = 50.0 s ?
Express your answer in meters.

Hint 1. Area of the region under the v vs. t curve

The region under the v vs. t curve between t = 40.0 s and t = 50.0 s is a triangle of base 10.0 s and
height 2.00 m/ s , as shown in the figure.

https://session.masteringphysics.com/myct/assignmentPrintView?assignmentID=7287775 Page 47 of 100


Taller de Autoaprendizaje No 1a 4/20/19, 4)33 PM

ANSWER:

∆x 3 = 10 m

ANSWER:

∆x = 75 m

Correct

Part C
What is the average acceleration a a v of the particle over the first 20.0 seconds?

Express your answer in meters per second per second.

Hint 1. Definition and graphical interpretation of average acceleration

The average acceleration a a v of a particle that travels along a straight line in a time interval ∆t is the ratio of the
change in velocity ∆v experienced by the particle to the time interval ∆t , or

∆v
aa v =
∆t
.
In a v vs. t graph, then, the average acceleration equals the slope of the line connecting the two points
representing the initial and final velocities.

Hint 2. Slope of a line

The slope m of a line from point A, of coordinates (x A , yA ), to point B, of coordinates (x B , yB ), is equal to the
"rise" over the "run," or
y B −y A
m= x B −x A .

ANSWER:

a a v = 0.075 m/ s 2

Correct
The average acceleration of a particle between two instants of time is the slope of the line connecting the two
corresponding points in a v vs. t graph.

Part D
t = 45.0 s
https://session.masteringphysics.com/myct/assignmentPrintView?assignmentID=7287775 Page 48 of 100
Taller de Autoaprendizaje No 1a 4/20/19, 4)33 PM

What is the instantaneous acceleration a of the particle at t = 45.0 s ?

Hint 1. Graphical interpretation of instantaneous acceleration


The acceleration of a particle at any given instant of time or at any point in its path is called the instantaneous
acceleration. If the v vs. t graph of the particle's motion is known, you can directly determine the instantaneous
acceleration at any point on the curve. The instantaneous acceleration at any point is equal to the slope of the
line tangent to the curve at that point.

Hint 2. Slope of a line


The slope m of a line from point A, of coordinates (x A , yA ), to point B, of coordinates (x B , yB ), is equal to the
"rise" over the "run," or
y B −y A
m= x B −x A .

ANSWER:

1 m/ s 2

0.20 m/ s 2

a= -0.20 m/ s 2

0.022 m/ s 2

-0.022 m/ s 2

Correct
The instantaneous acceleration of a particle at any point on a v vs. t graph is the slope of the line tangent to
the curve at that point. Since in the last 10 seconds of motion, between t = 40.0 s and t = 50.0 s , the curve
is a straight line, the tangent line is the curve itself. Physically, this means that the instantaneous acceleration
of the particle is constant over that time interval. This is true for any motion where velocity increases linearly
with time. In the case at hand, can you think of another time interval in which the acceleration of the particle is
constant?

Now that you have reviewed how to plot variables as a function of time, you can use the same technique and draw an
acceleration vs. time graph, that is, the graph of (instantaneous) acceleration as a function of time. As usual in these types of
graphs, time t is plotted on the horizontal axis, while the vertical axis is used to indicate acceleration a .

Part E
Which of the graphs shown below is the correct acceleration vs. time plot for the motion described in the previous parts?

https://session.masteringphysics.com/myct/assignmentPrintView?assignmentID=7287775 Page 49 of 100


Taller de Autoaprendizaje No 1a 4/20/19, 4)33 PM

Hint 1. How to approach the problem


Recall that whenever velocity increases linearly with time, acceleration is constant. In the example here, the
particle's velocity increases linearly with time in the first 20.0 s of motion. In the second 20.0 s , the particle's
velocity is constant, and then it decreases linearly with time in the last 10 s . This means that the particle's
acceleration is constant over each time interval, but its value is different in each interval.

Hint 2. Find the acceleration in the first 20 s


What is a 1 , the particle's acceleration in the first 20 s of motion, between t = 0.0 s and t = 20.0 s ?
Express your answer in meters per second per second.

Hint 1. Constant acceleration


Since we have already determined that in the first 20 s of motion the particle's acceleration is constant, its
constant value will be equal to the average acceleration that you calculated in Part C.

ANSWER:

a 1 = 0.075 m/ s 2

https://session.masteringphysics.com/myct/assignmentPrintView?assignmentID=7287775 Page 50 of 100


Taller de Autoaprendizaje No 1a 4/20/19, 4)33 PM

Correct

Hint 3. Find the acceleration in the second 20 s


What is a 2 , the particle's acceleration in the second 20 s of motion, between t = 20.0 s and t = 40.0 s ?
Express your answer in meters per second per second.

Hint 1. Constant velocity

In the second 20 s of motion, the particle's velocity remains unchanged. This means that in this time
interval, the particle does not accelerate.

ANSWER:

a 2 = 0 m/ s 2

Correct

Hint 4. Find the acceleration in the last 10 s

What is a 3 , the particle's acceleration in the last 10 s of motion, between t = 40.0 s and t = 50.0 s ?
Express your answer in meters per second per second.

Hint 1. Constant acceleration

Since we have already determined that in the last 10 s of motion the particle's acceleration is constant, its
constant value will be equal to the instantaneous acceleration that you calculated in Part D.

ANSWER:

a 3 = -0.20 m/ s 2

Correct

ANSWER:

https://session.masteringphysics.com/myct/assignmentPrintView?assignmentID=7287775 Page 51 of 100


Taller de Autoaprendizaje No 1a 4/20/19, 4)33 PM

Graph A

Graph B

Graph C

Graph D

Correct
In conclusion, graphs of velocity as a function of time are a useful representation of straight-line motion. If read
correctly, they can provide you with all the information you need to study the motion.

Velocity and Acceleration of a Power Ball

Learning Goal:

To understand the distinction between velocity and acceleration with the use of motion diagrams.
In common usage, velocity and acceleration both can imply having considerable speed. In physics, they are sharply defined
concepts that are not at all synonymous. Distinguishing clearly between them is a prerequisite to understanding motion.
Moreover, an easy way to study motion is to draw a motion diagram, in which the position of the object in motion is sketched
at several equally spaced instants of time, and these sketches (or snapshots) are combined into one single picture.

In this problem, we make use of these concepts to study the motion of a power ball. This discussion assumes that we have
already agreed on a coordinate system from which to measure the position r (t) ⃗ (also called the position vector) of objects as
⃗ and a (t)
a function of time. Let v(t) ⃗ be velocity and acceleration, respectively.

Consider the motion of a power ball that is dropped on the floor and bounces back. In the following questions, you will
describe its motion at various points in its fall in terms of its velocity and acceleration.

Part A
You drop a power ball on the floor. The motion diagram of the ball is sketched in the figure . Indicate whether the
magnitude of the velocity of the ball is increasing, decreasing, or not changing.

https://session.masteringphysics.com/myct/assignmentPrintView?assignmentID=7287775 Page 52 of 100


Taller de Autoaprendizaje No 1a 4/20/19, 4)33 PM

Hint 1. Velocity and displacement vectors

By definition, the velocity is the ratio of the distance traveled to the interval of time taken. If you interpret the
vector displacement ∆r as ⃗ the distance traveled by the ball, the length of v is⃗ directly proportional to the length

of ∆r . Since the length of displacement vectors is increasing, so is the length of velocity vectors.

ANSWER:

increasing

decreasing

not changing

Correct
While the ball is in free fall, the magnitude of its velocity is increasing, so the ball is accelerating.

Part B
Since the length of v is⃗ directly proportional to the length of ∆r , the
⃗ vector connecting each dot to the next could
represent velocity vectors as well as displacement vectors, as shown in the figure here . Indicate whether the velocity
and acceleration of the ball are, respectively, positive (upward), negative, or zero.

Use P, N, and Z for positive (upward), negative, and zero, respectively. Separate the letters for velocity and
acceleration with a comma.

https://session.masteringphysics.com/myct/assignmentPrintView?assignmentID=7287775 Page 53 of 100


Taller de Autoaprendizaje No 1a 4/20/19, 4)33 PM

Hint 1. Acceleration vector

The acceleration is defined as the ratio of the change in velocity to the interval of time, and its direction is given
by the quantity ∆v = ⃗ v(t⃗ 2 ) − v(t⃗ 1 ) , which represents the change in velocity that occurs in the interval of time
∆t = t2 − t1 .

ANSWER:

N,N

Correct

Part C
Now, consider the motion of the power ball once it bounces upward. Its motion diagram is shown in the figure here .
Indicate whether the magnitude of the velocity of the ball is increasing, decreasing, or not changing.

https://session.masteringphysics.com/myct/assignmentPrintView?assignmentID=7287775 Page 54 of 100


Taller de Autoaprendizaje No 1a 4/20/19, 4)33 PM

Hint 1. Velocity and displacement vectors

By definition, the velocity is the ratio of the distance traveled to the interval of time taken. If you interpret the
vector displacement ∆r as ⃗ the distance traveled by the ball, the length of v is⃗ directly proportional to the length

of ∆r . Since the length of displacement vectors is decreasing, so is the length of velocity vectors.

ANSWER:

increasing

decreasing

not changing

Correct
Since the magnitude of the velocity of the ball is decreasing, the ball must be accelerating (specifically, slowing
down).

Part D
The next figure shows the velocity vectors corresponding to the upward motion of the power ball. Indicate whether its
velocity and acceleration, respectively, are positive (upward), negative, or zero.

Use P, N, and Z for positive (upward), negative, and zero, respectively. Separate the letters for velocity and
acceleration with a comma.

https://session.masteringphysics.com/myct/assignmentPrintView?assignmentID=7287775 Page 55 of 100


Taller de Autoaprendizaje No 1a 4/20/19, 4)33 PM

Hint 1. Acceleration vector

The acceleration is defined as the ratio of the change in velocity to the interval of time, and its direction is given
by the quantity ∆v = ⃗ v(t⃗ 2 ) − v(t⃗ 1 ) , which represents the change in velocity that occurs in the interval of time
∆t = t2 − t1 .

ANSWER:

P,N

Correct

Part E
The power ball has now reached its highest point above the ground and starts to descend again. The motion diagram
representing the velocity vectors is the same as that after the initial release, as shown in the figure of Part B. Indicate
whether the velocity and acceleration of the ball at its highest point are positive (upward), negative, or zero.

Use P, N, and Z for positive (upward), negative, and zero, respectively. Separate the letters for velocity and
acceleration with a comma.

Hint 1. Velocity as a continuous function of time


In Part D you found that the velocity of the ball is positive during the upward motion. Once the ball starts its
descent, its velocity is negative, as you found in Part B. Since velocity changes continuously in time, it has to be
zero at some point along the path of the ball.

Hint 2. Acceleration as a continuous function of time

https://session.masteringphysics.com/myct/assignmentPrintView?assignmentID=7287775 Page 56 of 100


Taller de Autoaprendizaje No 1a 4/20/19, 4)33 PM

In Part D, you found that the acceleration of the ball is negative and constant during the upward motion, as well
as once the ball has started its descent, which you found in Part B. Since acceleration is a continuous function of
time, it has to be negative at the highest point along the path as well.

ANSWER:

Z,N

Correct
These examples should show you that the velocity and acceleration can have opposite or similar signs or that
one of them can be zero while the other has either sign. Try hard to think carefully about them as distinct
physical quantities when working with kinematics.

One-Dimensional Kinematics with Constant Acceleration

Learning Goal:

To understand the meaning of the variables that appear in the equations for one-dimensional kinematics with constant
acceleration.
Motion with a constant, nonzero acceleration is not uncommon in the world around us. Falling (or thrown) objects and cars
starting and stopping approximate this type of motion. It is also the type of motion most frequently involved in introductory
kinematics problems.

The kinematic equations for such motion can be written as

x(t) = xi + v i t + 12 at2 ,
v(t) = vi + at,
where the symbols are defined as follows:

x(t) is the position of the particle;


x i is the initial position of the particle;
v(t) is the velocity of the particle;
vi is the initial velocity of the particle;
a is the acceleration of the particle.

In anwering the following questions, assume that the acceleration is constant and nonzero: a ≠ 0.

Part A
The quantity represented by x is a function of time (i.e., is not constant).

ANSWER:

https://session.masteringphysics.com/myct/assignmentPrintView?assignmentID=7287775 Page 57 of 100


Taller de Autoaprendizaje No 1a 4/20/19, 4)33 PM

true

false

Correct

Part B
The quantity represented by x i is a function of time (i.e., is not constant).
ANSWER:

true

false

Correct
Recall that x i represents an initial value, not a variable. It refers to the position of an object at some initial
moment.

Part C
The quantity represented by vi is a function of time (i.e., is not constant).

ANSWER:

true

false

Correct

Part D
The quantity represented by v is a function of time (i.e., is not constant).
ANSWER:

https://session.masteringphysics.com/myct/assignmentPrintView?assignmentID=7287775 Page 58 of 100


Taller de Autoaprendizaje No 1a 4/20/19, 4)33 PM

true

false

Correct
The velocity v always varies with time when the linear acceleration is nonzero.

Part E
Which of the given equations is not an explicit function of t and is therefore useful when you don't know or don't need the
time?

ANSWER:

x = xi + v i t + 12 at2
v = vi + at
v2 = v2i + 2a(x − x i )

Correct

Part F
A particle moves with constant acceleration a . The expression vi + at represents the particle's velocity at what instant
in time?
ANSWER:

only at time t =0

only at the "initial" time

when a time t has passed since the particle's velocity was vi

Correct

More generally, the equations of motion can be written as

2
x(t) = i + i ∆t + 1 a (∆t
https://session.masteringphysics.com/myct/assignmentPrintView?assignmentID=7287775 Page 59 of 100
Taller de Autoaprendizaje No 1a 4/20/19, 4)33 PM

x(t) = xi + v i ∆t + 12 a (∆t) 2
and

v(t) = vi + a ∆t.
Here ∆t is the time that has elapsed since the beginning of the particle's motion, that is, ∆t = t − ti , where t is the current
time and ti is the time at which we start measuring the particle's motion. The terms x i and vi are, respectively, the position
and velocity at t = ti . As you can now see, the equations given at the beginning of this problem correspond to the case
ti = 0 , which is a convenient choice if there is only one particle of interest.
To illustrate the use of these more general equations, consider the motion of two particles, A and B. The position of particle A
2
depends on time as x A (t) = x i + vi t + (1/ 2)at . That is, particle A starts moving at time t = tiA = 0 with velocity
viA = vi , from x iA = x i . At time t = t1 , particle B has twice the acceleration, half the velocity, and the same position that
particle A had at time t = 0.

Part G
What is the equation describing the position of particle B?

Hint 1. How to approach the problem


The general equation for the distance traveled by particle B is
xB (t) = xiB + v iB ∆t + 12 a B (∆t) 2 ,
or

xB (t) = xB (t = t1 ) + v B (t = t1 )(t − t1 ) + 12 a B (t − t1 ) 2 ,
since ∆t = t − t1 is a good choice for B. From the information given, deduce the correct values of the
constants that go into the equation for x B (t) given here, in terms of A's constants of motion.

ANSWER:

xB (t) = xi + 2v i t + 14 at2
x B (t) = x i + 0.5vi t + at2

xB (t) = xi + 2v i (t + t1 ) + 14 a(t + t1 ) 2
2
x B (t) = x i + 0.5vi (t + t1 ) + a(t + t1 )

xB (t) = xi + 2v i (t − t1 ) + 14 a(t − t1 ) 2
2
x B (t) = x i + 0.5vi (t − t1 ) + a(t − t1 )

Correct

https://session.masteringphysics.com/myct/assignmentPrintView?assignmentID=7287775 Page 60 of 100


Taller de Autoaprendizaje No 1a 4/20/19, 4)33 PM

Part H
At what time does the velocity of particle B equal that of particle A?

Hint 1. Velocity of particle A


Type an expression for particle A's velocity as a function of time.

Express your answer in terms of t and some or all of the variables x i , vi , and a .

Hint 1. How to approach this part


Look at the general expression for v(t) given in the problem introduction.

ANSWER:

vA (t) = vi + at

Hint 2. Velocity of particle B


Type an expression for particle B's velocity as a function of time.
Express your answer in terms of t and some or all of the variables t1 , x i , vi , and a .

Hint 1. How to approach this part

The general expression for vB (t) is

vB (t) = vB (t = t1 ) + a B (t − t1 ).
From the information given, deduce the correct values of the constants that go into this equation in terms
of particle A's constants of motion.

ANSWER:

vB (t) = 0.5vi + 2a(t − t1 )

ANSWER:

https://session.masteringphysics.com/myct/assignmentPrintView?assignmentID=7287775 Page 61 of 100


Taller de Autoaprendizaje No 1a 4/20/19, 4)33 PM

vi
t = t1 + 4a
vi
t = 2t1 + 2a
vi
t = 3t1 + 2a

The two particles never have the same velocity.

Correct

PSS 2.1 Motion with Constant Acceleration

Learning Goal:

To practice Problem-Solving Strategy 2.1 Motion with Constant Acceleration.


Cheetahs, the fastest of the great cats, can reach 50.0 miles/ hour in 2.22 {\rm \rm {s}} starting from rest. Assuming that
they have constant acceleration throughout that time, find their acceleration in meters per second squared.

Problem-Solving Strategy: Motion with constant acceleration

IDENTIFY the relevant concepts:


In most straight-line motion problems, you can use the constant-acceleration equations. Occasionally, however, you will
encounter a situation in which the acceleration isn’t constant. In such a case, you’ll need a different approach.

SET UP the problem using the following steps:

1. First, decide where the origin of coordinates is and which axis direction is positive. It is often easiest to place
the particle at the origin at time t = 0; then x_0 = 0. It helps to make a motion diagram showing the coordinates
and some later positions of the particle.
2. Keep in mind that your choice of the positive axis direction automatically determines the positive direction for x
velocity and x acceleration. If \texttip{x}{x} is positive to the right of the origin, then \texttip{v_{\mit x}}{v_x} and
\texttip{a_{\mit x}}{a_x} are also positive toward the right.
3. Restate the problem in words, and then translate it into symbols and equations.
4. Make a list of known and unknown quantities such as \texttip{x}{x}, \texttip{x_{\rm 0}}{x_0}, \texttip{v_{\mit x}}
{v_x}, \texttip{v_{0x}}{v_0x}, \texttip{a_{\mit x}}{a_x}, and \texttip{t}{t}. Write down the values of the known
quantities, and decide which of the unknowns are the target variables. Look for implicit information.

EXECUTE the solution as follows:


Choose an equation from the following list

v_x = v_{0x} + a_xt

\large{x = x_0 + v_{0x}t + \frac{1}{2} a_x t^2}

v_x^2 = v_{0x}^2 + 2 a_x (x - x_0)

\large{x - x_0 = \left( \frac {v_{0x} + v_x}{2}\right) t}

https://session.masteringphysics.com/myct/assignmentPrintView?assignmentID=7287775 Page 62 of 100


Taller de Autoaprendizaje No 1a 4/20/19, 4)33 PM

that contains only one of the target variables. Solve this equation for the target variable, using symbols only. Then, substitute
the known values and compute the value of the target variable. Sometimes you will have to solve two simultaneous
equations for two unknown quantities.

EVALUATE your answer:


Take a hard look at your results to see whether they make sense. Are they within the general range of values you expected?

IDENTIFY the relevant concepts


This problem involves the motion of an object, the cheetah, whose acceleration is assumed constant. Thus, the equations
given in this strategy apply.

SET UP the problem using the following steps

Part A
Which of the following sketches and choice of coordinate axis best describe the physical situation presented in this
problem?

ANSWER:

Correct

https://session.masteringphysics.com/myct/assignmentPrintView?assignmentID=7287775 Page 63 of 100


Taller de Autoaprendizaje No 1a 4/20/19, 4)33 PM

Part B
The next step is to translate the problem statement from words into symbols. Which of the following is an appropriate
restatement of the problem, "Cheetahs, the fastest of the great cats, can reach 50.0 {\rm \rm {miles/hour}} in 2.22 {\rm
\rm {s}} starting from rest. Assuming that they have constant acceleration throughout that time, find their acceleration in
meters per second squared."

Hint 1. Find the initial velocity using implicit information


The problem states that the cheetah starts running from rest. What is the initial velocity \texttip{v_{0x}}{v_0x} of
the cheetah?
Enter your answer in meters per second.
ANSWER:

\texttip{v_{0x}}{v_0x} = 0 \rm {m/s}

Hint 2. The condition for the equations of motion presented in this problem

The equations presented in the strategy above only apply to situations involving motion under constant
acceleration.

ANSWER:

Cheetahs can reach v_{0x}= 50.0 {\rm \rm {miles/hour}} in \texttip{t}{t} = 2.22 {\rm \rm {s}} starting from v_x =
0. What is \texttip{a_{\mit x}}{a_x}?

Cheetahs can reach \texttip{v_{\mit x}}{v_x} = 50.0 {\rm \rm {miles/hour}} in \texttip{t}{t} = 2.22 {\rm \rm {s}}
starting from v_{0x} = 0. What is \texttip{a_{\mit x}}{a_x}?

Cheetahs can reach \texttip{v_{\mit x}}{v_x} = 50.0 {\rm \rm {miles/hour}} in \texttip{t}{t} = 2.22 {\rm \rm {s}}
starting from v_{0x} = 0. Assuming a = \rm{constant}, what is \texttip{a_{\mit x}}{a_x}?

Cheetahs can reach v_{0x}= 50.0 {\rm \rm {miles/hour}} in \texttip{t}{t} = 2.22 {\rm \rm {s}} starting from v_x =
0. Assuming a = \rm{constant}, what is \texttip{a_{\mit x}}{a_x}?

https://session.masteringphysics.com/myct/assignmentPrintView?assignmentID=7287775 Page 64 of 100


Taller de Autoaprendizaje No 1a 4/20/19, 4)33 PM

Correct
Now you compile a list of known and unknown quantities. You can organize this information in a table as
shown below.
Known Unknown
x_0=0\;\rm m \texttip{x}{x}
\texttip{a_{\mit
v_{0x}=0\;\rm m/s
x}}{a_x}
\texttip{v_{\mit x}}{v_x} = 50.0 {\rm
_
\rm {miles/hour}}

\texttip{t}{t} = 2.22 {\rm \rm {s}} _

Keep in mind that your target variable is \texttip{a_{\mit x}}{a_x}.

EXECUTE the solution as follows

Part C
Finally, you are ready to answer the main question. Cheetahs, the fastest of the great cats, can reach 50.0 {\rm \rm
{miles/hour}} in 2.22 {\rm \rm {s}} starting from rest. Assuming that they have constant acceleration throughout that time,
find their acceleration in meters per second squared.

Enter your answer in meters per second squared to three significant figures.

Hint 1. Identify what equation to use

Which of the following equations would be the best to use when solving for \texttip{a_{\mit x}}{a_x}?
ANSWER:

\large{x - x_0 = \left(\frac{v_{0x} + v_x}{2}\right)t}

\large{x = x_0 + v_{0x}t + \frac{1}{2} a_x t^2}

v_x = v_{0x} + a_xt

v_x^2 = v_{0x}^2 + 2a_x(x - x_0)

Hint 2. Convert to SI units


How many meters per second are equivalent to 50.0 {\rm \rm {miles/hour}} ?
Enter your answer in meters per second to three significant figures.

Hint 1. The conversion factor from miles to meters

https://session.masteringphysics.com/myct/assignmentPrintView?assignmentID=7287775 Page 65 of 100


Taller de Autoaprendizaje No 1a 4/20/19, 4)33 PM

To convert miles to meters, use


1 \;\rm {mile} = 1609 \;\rm m.

ANSWER:

50.0 {\rm \rm {miles/hour}} = 22.3 \rm {m/s}

ANSWER:

\texttip{a_{\mit x}}{a_x} = 10.1 \rm {m/s^2}

Correct

EVALUATE your answer

Part D
Imagine you looked up the accelerations of the following objects: snails, humans, Thomson's gazelles, the space shuttle,
Formula One race cars, and F-16 fighter jets. Which of the following statements about the acceleration of a cheetah
would you expect to be true?
ANSWER:

The acceleration of a cheetah is greater than the acceleration of a snail but less than the acceleration of a
human.

The acceleration of a cheetah is greater than the acceleration of a Formula One race car but less than the
acceleration of an F-16 fighter jet.

The acceleration of a cheetah is greater than the acceleration of a Thomson's gazelle but less than the
acceleration of the space shuttle during liftoff.

Correct
The acceleration of the space shuttle on takeoff is 29.4 \rm{m/s^2}. Thomson's gazelles can accelerate at
approximately half the rate of a cheetah, which is why they often become tasty snacks for the fast cats.

If you had solved for the acceleration of a cheetah and calculated a number greater than 29.4 \rm{m/s^2} or
smaller than 30 \rm{cm/s^2} (the acceleration of a snail), you most likely made an error and would want to
review your work.

Motion of Two Rockets

https://session.masteringphysics.com/myct/assignmentPrintView?assignmentID=7287775 Page 66 of 100


Taller de Autoaprendizaje No 1a 4/20/19, 4)33 PM

Learning Goal:
To learn to use images of an object in motion to determine position, velocity, and acceleration.
Two toy rockets are traveling in the same direction (taken to be the x axis). A diagram is shown of a time-exposure image
where a stroboscope has illuminated the rockets at the uniform time intervals indicated.

Part A
At what time(s) do the rockets have the same velocity?

Hint 1. How to determine the velocity

The diagram shows position, not velocity. You can't find instantaneous velocity from this diagram, but you can
determine the average velocity between two times \texttip{t_{\rm 1}}{t_1} and \texttip{t_{\rm 2}}{t_2}:

\large{v_{\rm avg}[t_1,t_2] = \frac{x(t_2) - x(t_1)}{t_2-t_1}}.

Note that no position values are given in the diagram; you will need to estimate these based on the distance
between successive positions of the rockets.

ANSWER:

https://session.masteringphysics.com/myct/assignmentPrintView?assignmentID=7287775 Page 67 of 100


Taller de Autoaprendizaje No 1a 4/20/19, 4)33 PM

at time t = 1 only

at time t = 4 only

at times t = 1 and t = 4

at some instant in time between t = 1 and t = 4

at no time shown in the figure

Correct

Part B
At what time(s) do the rockets have the same x position?
ANSWER:

at time t = 1 only

at time t = 4 only

at times t = 1 and t = 4

at some instant in time between t = 1 and t = 4

at no time shown in the figure

Correct

Part C
At what time(s) do the two rockets have the same acceleration?

Hint 1. How to determine the acceleration


The velocity is related to the spacing between images in a stroboscopic diagram. Since acceleration is the rate at
which velocity changes, the acceleration is related to the how much this spacing changes from one interval to the
next.

https://session.masteringphysics.com/myct/assignmentPrintView?assignmentID=7287775 Page 68 of 100


Taller de Autoaprendizaje No 1a 4/20/19, 4)33 PM

ANSWER:

at time t = 1 only

at time t = 4 only

at times t = 1 and t = 4

at some instant in time between t = 1 and t = 4

at no time shown in the figure

Correct

Part D
The motion of the rocket labeled A is an example of motion with uniform (i.e., constant) __________.

ANSWER:

and nonzero acceleration

velocity

position

time

Correct

Part E
The motion of the rocket labeled B is an example of motion with uniform (i.e., constant) __________.
ANSWER:

and nonzero acceleration

velocity

position

time

https://session.masteringphysics.com/myct/assignmentPrintView?assignmentID=7287775 Page 69 of 100


Taller de Autoaprendizaje No 1a 4/20/19, 4)33 PM

Correct

Part F
At what time(s) is rocket A ahead of rocket B?

Hint 1. Use the diagram

You can answer this question by looking at the diagram and identifying the time(s) when rocket A is to the right of
rocket B.

ANSWER:

before t = 1 only

after t = 4 only

before t = 1 and after t = 4

between t = 1 and t = 4

at no time(s) shown in the figure

Correct

± Rocket Height

A rocket, initially at rest on the ground, accelerates straight upward from rest with constant acceleration 53.9 {\rm m/s^2} .
The acceleration period lasts for time 5.00 {\rm s} until the fuel is exhausted. After that, the rocket is in free fall.

Part A
Find the maximum height \texttip{y_{\rm max}}{y_max} reached by the rocket. Ignore air resistance and assume a
constant acceleration due to gravity equal to 9.80 \rm{m/s^2} .
Write your answer numerically in units of meters.

Hint 1. How to approach the problem


Divide the upward motion into two parts: first the fueled motion, and then the motion under the influence of gravity
alone. Find the height reached over the course of the fueled motion, and then calculate the additional height
achieved during the second part of the motion. Putting these two distances together will give you the maximum
height reached by the rocket.

https://session.masteringphysics.com/myct/assignmentPrintView?assignmentID=7287775 Page 70 of 100


Taller de Autoaprendizaje No 1a 4/20/19, 4)33 PM

Hint 2. Find the height reached during the fueled part of the motion
Find the height \texttip{y_{\rm fuel}}{y_fuel} above the ground at which the rocket exhausts its fuel.
Answer numerically in units of meters.

Hint 1. Knowns and unknowns


At the instant that the rocket takes off, take time t_0=0 and the initial position y_0=0. Let the final values of
the variables correspond to those at which the rocket runs out of fuel. Clearly, the final height
\texttip{y_{\rm fuel}}{y_fuel} for the fueled portion of the flight and the associated final
velocity\texttip{v_{\rm fuel}}{v_fuel} are not given.

Let us denote other quantities as follows: \texttip{t_{\rm fuel}}{t_fuel} is the time that the rocket travels
before it runs out of fuel; \texttip{v_{\rm 0,fuel}}{v_0,fuel} is the rocket's initial velocity; and \texttip{a_{\rm
fuel}}{a_fuel} is the rocket's net acceleration during the fueled portion of its flight. Which of the these
quantities are known?

Check all that apply.

Hint 1. What is the initial velocity?


What is the initial velocity \texttip{v_{\rm 0,fuel}}{v_0,fuel} for the fueled part of the motion?
Give your answer numerically.

ANSWER:

\texttip{v_{\rm 0,fuel}}{v_0,fuel} = 0 \rm m/s

ANSWER:

\texttip{t_{\rm fuel}}{t_fuel}

\texttip{v_{\rm 0,fuel}}{v_0,fuel}

\texttip{a_{\rm fuel}}{a_fuel}

Hint 2. Determine which kinematic equation to use


Choose the kinematic equation that makes the solution straighforward, that is, the one that contains the
variable you are solving for and in which all of the other quantities are known.
ANSWER:

https://session.masteringphysics.com/myct/assignmentPrintView?assignmentID=7287775 Page 71 of 100


Taller de Autoaprendizaje No 1a 4/20/19, 4)33 PM

v=v_0+at

\large{y=y_0+v_0t+\frac12at^2}

v^2=v_0^2+2a(y-y_0)

\large{y-y_0=\left( \frac{v_0+v}{2} \right) t}

ANSWER:

\texttip{y_{\rm fuel}}{y_fuel} = 674 \rm m

Incorrect; Try Again

Hint 3. Find the initial velocity, the final velocity, and the acceleration for the "free-fall" part of the motion
What are \texttip{v_{\rm 0,grav}}{v_0,grav}, \texttip{v_{\rm f,grav}}{v_f,grav}, and \texttip{a_{\rm grav}}{a_grav} for
the second part of the motion?
Write your answer numerically in the order v_{0, \rm grav},v_{\rm f, grav},a_{\rm grav}, separated by
commas as shown, in SI units.

Hint 1. What is the initial velocity?

When the rocket runs out of fuel, its acceleration changes abruptly, but its velocity changes continuously.
Therefore, the rocket's initial velocity \texttip{v_{\rm 0,grav}}{v_0,grav} for the second part of the flight is
just its velocity at the moment the engine runs out of fuel. What, then, is \texttip{v_{\rm 0,grav}}{v_0,grav}?
Give your answer numerically.

Hint 1. Find the velocity when the engine runs out of fuel
For the fueled part of the motion, you know that the initial velocity is given by v_{0, \rm fuel} =0, the
acceleration by a_{\rm fuel} =53.9 {\rm m/s^2} , and the time of fueled flight by t_{\rm fuel} =5.00
{\rm s} . You also determined the height \texttip{y_{\rm fuel}}{y_fuel} in Part A.2.
Choose a kinematic equation that you could use to find \texttip{v_{\rm f,fuel}}{v_f,fuel}, the velocity
at the end of the fueled motion.

1. v=v_0+at
2. y=y_0+v_0t+(1/2)at^2
3. v^2=v_0^2+2a(y-y_0)
4. \large{y-y_0=\left( \frac{v_0+v}{2} \right) t}
Choose one letter corresponding to the equation you have chosen (even though there is
more than one correct answer).
ANSWER:

https://session.masteringphysics.com/myct/assignmentPrintView?assignmentID=7287775 Page 72 of 100


Taller de Autoaprendizaje No 1a 4/20/19, 4)33 PM

ANSWER:

\texttip{v_{\rm 0,grav}}{v_0,grav} = 270 \rm m/s

Hint 2. What is the acceleration?


What value should you use for the acceleration \texttip{a_{\rm grav}}{a_grav}? Keep in mind that the
direction is important, since the acceleration due to gravity is slowing down the rocket as it continues its
ascent.

Give your answer numerically.


ANSWER:

\texttip{a_{\rm grav}}{a_grav} = -9.80 \rm m/s^2

Hint 3. What is the final velocity?

What is the velocity \texttip{v_{\rm f,grav}}{v_f,grav} of the rocket when it reaches its maximum height?
Note that the rocket has just ended its ascent and is about to begin its descent. What is its velocity at this
instant?
Give your answer numerically.
ANSWER:

\texttip{v_{\rm f,grav}}{v_f,grav} = 0 \rm m/s

ANSWER:

v_{0, \rm grav},v_{\rm f,grav},a_{\rm grav} = 270,0,-9.80 SI units

Hint 4. Determine which kinematic equation to use


Choose the kinematic equation that makes the solution straightforward, that is, the one that contains the variable

https://session.masteringphysics.com/myct/assignmentPrintView?assignmentID=7287775 Page 73 of 100


Taller de Autoaprendizaje No 1a 4/20/19, 4)33 PM

you are solving for and for which all of the other quantities are known.
ANSWER:

v=v_0+at

y=y_0+v_0t+(1/2)at^2

v^2=v_0^2+2a(y-y_0)

\large{y-y_0=\left( \frac{v_0+v}{2} \right) t}

ANSWER:

\texttip{y_{\rm max}}{y_max} = 4380 \rm m

Answer Requested

A Flea in Flight

In this problem, you will apply kinematic equations to a jumping flea. Take the magnitude of free-fall acceleration to be 9.80
{\rm m/s^2} . Ignore air resistance.

Part A
A flea jumps straight up to a maximum height of 0.550 {\rm m} . What is its initial velocity \texttip{v_{\rm 0}}{v_0} as it
leaves the ground?
Express your answer in meters per second to three significant figures.

Hint 1. Finding the knowns and unknowns


Take the positive y direction to be upward, the y coordinate of the initial position of the flea to be y_0 = 0, and
denote the final height of the flea by \texttip{y_{\rm 1}}{y_1}, whose value 0.550 {\rm m} you know. Let \texttip{t}{t}
be the duration of the flea's leap to its maximum height, \texttip{v_{\rm 0}}{v_0} its initial velocity, \texttip{v_{\rm 1}}
{v_1} its final velocity (at maximum height), and \texttip{a_{\rm y}}{a_{\rm y}} its (constant) acceleration. Which of
the following quantities is/are known?
Check all that apply.

Hint 1. The number of known quantities


Typically, you need to know the values of four variables in order to solve any of the kinematic equations,
because they contain five variables each, with the exception of v_1=v_0 + a_{\rm y}t, which contains only
four variables, in which case you would need to know the values of only three of these variables. Since we

https://session.masteringphysics.com/myct/assignmentPrintView?assignmentID=7287775 Page 74 of 100


Taller de Autoaprendizaje No 1a 4/20/19, 4)33 PM

may place the flea at any point of the y axis to begin its jump, we have conveniently assumed that
\texttip{y_{\rm 0}}{y_0} is equal to 0.

Hint 2. What is the flea's velocity at its maximum height?


What is the velocity \texttip{v_{\rm 1}}{v_1} of the flea at its maximum height of y_1= 0.550 {\rm m} ?
Express your answer in meters per second to three significant figures.
ANSWER:

\texttip{v_{\rm 1}}{v_1} = 0 \rm m/s

ANSWER:

\texttip{t}{t}

\texttip{v_{\rm 0}}{v_0}

\texttip{v_{\rm 1}}{v_1}

\texttip{a_{\rm y}}{a_{\rm y}}

Hint 2. Determine which kinematic equation to use

Decide which kinematic equation makes the solution of this problem easiest. That is, look for an equation that
contains the variable you are solving for and in which all the other variables are known.
ANSWER:

v_1=v_0+a_{\rm y}t

\large{y_1=y_0+v_0t+\frac{1}{2}a_{\rm y}t^2}

v_1^2=v_0^2+2a_{\rm y}(y_1-y_0)

\large{y_1-y_0=\left( \frac{v_0+v_1}{2} \right) t}

Hint 3. Some algebra help


You have determined that the simplest equation to use is
v_1^2 = v_0^2 + 2a_{\rm y}(y_1-y_0).
To solve for \texttip{v_{\rm 0}}{v_0}, you must first subtract the term 2a_{\rm y}(y_1-y_0) from both sides of the
equation, and then take the square root of both sides. Keep in mind that the acceleration is negative.

ANSWER:

https://session.masteringphysics.com/myct/assignmentPrintView?assignmentID=7287775 Page 75 of 100


Taller de Autoaprendizaje No 1a 4/20/19, 4)33 PM

v_0 = 3.28 \rm m/s

Correct

Part B
How long is the flea in the air from the time it jumps to the time it hits the ground?
Express your answer in seconds to three significant figures.

Hint 1. How to approach the problem


One approach is to find the time it takes for the flea to go from the ground to its maximum height, and then find
the time it takes for the flea to fall from its maximum height to the ground. The subsequent hints will guide you
through this approach.

Hint 2. Find the time from the ground to the flea's maximum height
What is the time \texttip{t_{\rm up}}{t_up} it takes the flea to go from the ground (y_0=0\;\rm m, \texttip{v_{\rm 0}}
{v_0}) to its maximum height (y_1= 0.550 {\rm m} , v_1=0\;\rm m)?
Express your answer in seconds to three significant figures.
ANSWER:

\texttip{t_{\rm up}}{t_up} = 0.335 \rm s

Hint 3. Find the time from the flea's maximum height to the ground

What is the time \texttip{t_{\rm down}}{t_down} that it takes for the flea to fall from its maximum height (y_0=
0.550 {\rm m} , v_0=0\;\rm m) to the ground (y_1= 0\;\rm m)?
Express your answer in seconds to three significant figures.
ANSWER:

t_{\rm down} = 0.335 \rm s

ANSWER:

time in air = 0.670 \rm s

https://session.masteringphysics.com/myct/assignmentPrintView?assignmentID=7287775 Page 76 of 100


Taller de Autoaprendizaje No 1a 4/20/19, 4)33 PM

Correct
Notice that the time for the flea to rise to its maximum height is equal to the time it takes for it to fall from that
height back to the ground. This is a general feature of projectile motion (any motion with a = -g) when air
resistance is neglected and the landing point is at the same height as the launch point.

There is also a way to find the total time in the air in one step: just use

\large{y=y_0+v_0t+\frac{1}{2}a_{\rm y}t^2}

and realize that you are looking for the value of \texttip{t}{t} for which y=y_0.

± The Graph of a Sports Car's Velocity

The graph in the figure shows the velocity \texttip{v}{v} of a sports car as a
function of time \texttip{t}{t}. Use the graph to answer the following questions.

Part A
Find the maximum velocity \texttip{v_{\rm max}}{v_max} of the car during the ten-second interval depicted in the graph.

Express your answer in meters per second to the nearest integer.

Hint 1. How to approach the problem


Because the graph displays the car's velocity at each moment in time, the maximum velocity of the car can be
found simply by locating the maximum value of the velocity on the graph.

ANSWER:

\texttip{v_{\rm max}}{v_max} = 55 \rm m/s

Correct

https://session.masteringphysics.com/myct/assignmentPrintView?assignmentID=7287775 Page 77 of 100


Taller de Autoaprendizaje No 1a 4/20/19, 4)33 PM

Part B
During which time interval is the acceleration positive?

Indicate the best answer.

Hint 1. Finding acceleration from the graph

Recall that acceleration is the rate of change of velocity with respect to time. Therefore, on this graph of velocity
vs. time, acceleration is the slope of the graph. Recall that the slope \texttip{m}{m} is defined by m=\Delta y/\Delta
x for a graph of \texttip{y}{y} vs. \texttip{x}{x}, or m=\Delta v/\Delta t in this case. If the graph is increasing from left
to right, then the slope is positive.

ANSWER:

t=0\;{\rm s} to t=6\;{\rm s}

t=0\;{\rm s} to t=4\;{\rm s}

t=0\;{\rm s} to t=10\;{\rm s}

t=4\;{\rm s} to t=10\;{\rm s}

t=2\;{\rm s} to t=6\;{\rm s}

Correct

Part C
Find the maximum acceleration \texttip{a_{\rm max}}{a_max} of the car.

Express your answer in meters per second per second to the nearest integer.

Hint 1. How to approach the problem


The car's acceleration is the rate of change of the car's velocity \texttip{v}{v} with respect to time \texttip{t}{t}. In
this problem, the car's velocity is given graphically, so the car's acceleration at a given moment is found from the
slope of the \texttip{v}{v} vs. \texttip{t}{t} curve at that moment. If the \texttip{v}{v} vs. \texttip{t}{t} curve over some
time interval is represented by a straight line, the instantaneous acceleration anywhere in that interval is equal to
the slope of the line, that is, to the average acceleration over that time interval.
To find the maximum acceleration, find the value of the curve's greatest positive slope.

Hint 2. Find the final velocity on the interval with greatest acceleration
The slope of the curve is greatest during the first second of motion. The slope of the graph on this interval is
given by the change in velocity divided by the change in time over the interval from t=0 to t=1. At time t = 0\; {\rm
s}, the car's velocity v(0) is zero. Find the velocity \texttip{v\left(1\right)}{v(1)} of the car at time t=1\;{\rm s}.

https://session.masteringphysics.com/myct/assignmentPrintView?assignmentID=7287775 Page 78 of 100


Taller de Autoaprendizaje No 1a 4/20/19, 4)33 PM

Express your answer in meters per second to the nearest integer.


ANSWER:

\texttip{v\left(1\right)}{v(1)} = 30 \rm m/s

ANSWER:

\texttip{a_{\rm max}}{a_max} = 30 \rm m/s^2

Correct

Part D
Find the minimum magnitude of the acceleration \texttip{a_{\rm min}}{a_min} of the car.
Express your answer in meters per second per second to the nearest integer.

Hint 1. How to approach the problem


To find the minimum magnitude of the acceleration of the car, you must find the point where the absolute value of
the slope is smallest.

ANSWER:

\texttip{a_{\rm min}}{a_min} = 0 \rm m/s^2

Correct

Part E
Find the distance \texttip{d_{\rm 0,2}}{d_0,2} traveled by the car between t=0\;\rm s and t=2\; \rm s.

Express your answer in meters to the nearest integer.

Hint 1. How to approach the problem


In this problem, the car's velocity as a function of time is given graphically, so the distance traveled is represented
by the area under the \texttip{v}{v} vs. \texttip{t}{t} graph between t=0\;\rm s and t=2 \; \rm s.

Hint 2. Find the distance traveled in the first second

What is the distance \texttip{d_{\rm 0,1}}{d_0,1} traveled between t=0\;\rm s and t=1\;\rm s?

https://session.masteringphysics.com/myct/assignmentPrintView?assignmentID=7287775 Page 79 of 100


Taller de Autoaprendizaje No 1a 4/20/19, 4)33 PM

Express your answer in meters.

Hint 1. The area of a triangle


Observe that the region in question is a triangle
, whose area is therefore one-half the product of
the base and the height.

ANSWER:

d_0,1 = 15 \rm m

Hint 3. Find the distance traveled in the second second


What is the distance \texttip{d_{\rm 1,2}}{d_1,2} traveled between t=1\;\rm s andt=2\;\rm s?

Express your answer in meters.

Hint 1. The shape of the region


The region under the graph between 1 and 2 seconds can be seen as consisting of a rectangle and a
triangle.

https://session.masteringphysics.com/myct/assignmentPrintView?assignmentID=7287775 Page 80 of 100


Taller de Autoaprendizaje No 1a 4/20/19, 4)33 PM

ANSWER:

\texttip{d_{\rm 1,2}}{d_1,2} = 40 \rm m

ANSWER:

\texttip{d}{d} = 55 \rm m

Correct

Prelecture Video: Acceleration

Click Play to watch the video. Answer the ungraded questions in the video and the graded follow-up questions at right.

https://session.masteringphysics.com/myct/assignmentPrintView?assignmentID=7287775 Page 81 of 100


Taller de Autoaprendizaje No 1a 4/20/19, 4)33 PM

Part A
Which of the motion diagrams in the figure below best matches the motion of the bungee jumper shown in the video?

ANSWER:

Correct
Because the video only follows the free fall portion of the jump, the correct motion diagram shows constant
downward acceleration.

Part B
Which of the following best describes the acceleration of a bungee jumper during free fall?
ANSWER:

zero

negative

constant

positive

https://session.masteringphysics.com/myct/assignmentPrintView?assignmentID=7287775 Page 82 of 100


Taller de Autoaprendizaje No 1a 4/20/19, 4)33 PM

Correct
Free fall acceleration is constant and has a value of 9.8 \rm m/s^2.

Part C
Three cars drive around a perfectly circular track. The speedometers in the figure below show the speed of each car
over the same 10-\rm s time interval. Which of the cars are accelerating?

ANSWER:

Car A

Cars A and B

Cars A, B, and C

Correct
All three cars are accelerating because the directions of their velocity vectors change constantly as they drive
in a circle. In addition, the speeds of cars A and B are changing, which also indicates acceleration.

Part D
Four cars undergo acceleration as described by the data in the following table.

Final Velocity (\rm


Car Initial Velocity (\rm m/s) Time period (\rm s)
m/s)

A 2.0 11.0 3.0

B -5.0 3.0 2.0


C 1.0 -5.0 2.0
D 0.0 25.0 10.0

Rank the accelerations from most positive to most negative.


ANSWER:

https://session.masteringphysics.com/myct/assignmentPrintView?assignmentID=7287775 Page 83 of 100


Taller de Autoaprendizaje No 1a 4/20/19, 4)33 PM

Reset Help

most-positive acceleration most-negative acceleration

Car B Car A Car D Car C

The correct ranking cannot be determined.

Correct
The average accelerations, from most positive to most negative, are as follows: (B) 4 \rm m/s^2, (A) 3 \rm
m/s^2, (D) 2.5 \rm m/s^2, and (C) −3 \rm m/s^2.

Prelecture Video: Speed and Velocity

Click Play to watch the video. Answer the ungraded questions in the video and the graded follow-up questions at right.

Part A
Suppose you are hiking along a trail. Make a comparison between the magnitude of your displacement and your
distance traveled.

https://session.masteringphysics.com/myct/assignmentPrintView?assignmentID=7287775 Page 84 of 100


Taller de Autoaprendizaje No 1a 4/20/19, 4)33 PM

Check all that apply.


ANSWER:

The magnitude of your displacement must be less than your distance traveled.

The magnitude of your displacement must be equal to your distance traveled.

The magnitude of your displacement must be greater than your distance traveled.

The magnitude of your displacement can be greater than your distance traveled.

The magnitude of your displacement can be equal to your distance traveled.

The magnitude of your displacement can be less than your distance traveled.

Correct
The magnitude of your displacement represents your change in position. That is, the magnitude of your
displacement is how far you are from your starting location along a straight line.

Part B
Suppose a runner completes one lap around a 400-\rm m track in a time of 50 \rm s. Calculate the magnitude of the
average velocity of the runner.
Express your answer in meters per second to the nearest integer.

ANSWER:

0 \rm {m/s}

Correct

Part C
Suppose a runner completes one lap around a 400-\rm m track in a time of 50 \rm s. Calculate the average speed of the
runner.
Express your answer in meters per second to the nearest integer.

ANSWER:

8 \rm {m/s}

https://session.masteringphysics.com/myct/assignmentPrintView?assignmentID=7287775 Page 85 of 100


Taller de Autoaprendizaje No 1a 4/20/19, 4)33 PM

Correct
Average velocity is defined in terms of displacement, whereas average speed is defined in terms of total
distance traveled.

Part D
Consulting the graph shown in , determine the object's
average velocity over the time interval from 2 to 4
seconds.
Express your answer in meters per second to the
nearest integer.

ANSWER:

5 \rm m/s

Correct
Average velocity can be determined from the slope of a line connecting two points on a position versus time
graph.

Exercise 1.4

The density of lead is 11.3 {\rm g/cm^3} .

Part A
What is this value in kilograms per cubic meter?
ANSWER:

https://session.masteringphysics.com/myct/assignmentPrintView?assignmentID=7287775 Page 86 of 100


Taller de Autoaprendizaje No 1a 4/20/19, 4)33 PM

1.13×104 {\rm kg/m^3 }

Correct

Exercise 1.14

With a wooden ruler you measure the length of a rectangular piece of sheet metal to be 11 {\rm mm} . You use micrometer
calipers to measure the width of the rectangle and obtain the value 5.98 {\rm mm} . Give your answers to the following
questions to the correct number of significant figures.

Part A
What is the area of the rectangle?
ANSWER:

A = 66 {\rm mm}^{2}

Correct

Part B
What is the ratio of the rectangle's width to its length?

ANSWER:

\large{\frac{\rm width}{\rm length }} = 0.54

Correct

Part C
What is the perimeter of the rectangle?
ANSWER:

P = 34 {\rm mm}

Correct

https://session.masteringphysics.com/myct/assignmentPrintView?assignmentID=7287775 Page 87 of 100


Taller de Autoaprendizaje No 1a 4/20/19, 4)33 PM

Part D
What is the difference between the length and width?

ANSWER:

\Delta = 5 {\rm mm}

Correct

Part E
What is the ratio of the length to the width?
ANSWER:

\large{\frac{\rm length }{\rm width}} = 1.8

Correct

Exercise 1.18

Part A
How many kernels of corn does it take to fill a 3-{\rm L} soft drink bottle? (Take that about four kernels fill 1 {\rm cm^3} .)
Express your answer using two significant figures.

ANSWER:

1.2×104 kernels

Correct

Exercise 2.3

You normally drive on the freeway between San Diego and Los Angeles at an average speed of 105 {\rm km/h}, and the trip
takes 2 {\rm h} and 20 {\rm min}. On a Friday afternoon, however, heavy traffic slows you down and you drive the same
distance at an average speed of only 74.5 {\rm km/h} .

Part A

https://session.masteringphysics.com/myct/assignmentPrintView?assignmentID=7287775 Page 88 of 100


Taller de Autoaprendizaje No 1a 4/20/19, 4)33 PM

How much longer does the trip take?


ANSWER:

t = 57.3 {\rm min}

Correct

Exercise 2.9

A ball moves in a straight line (the x-axis). The graph in the figure
shows this ball's velocity as a function of time.

Part A
What is the ball's average velocity during the first 2.6 {\rm s} ?

Express your answer using two significant figures.


ANSWER:

v_{\rm av} = 2.2 {\rm m/s }

Correct

Part B
What is the ball's average speed during the first 2.6 {\rm s} ?

Express your answer using two significant figures.


ANSWER:

https://session.masteringphysics.com/myct/assignmentPrintView?assignmentID=7287775 Page 89 of 100


Taller de Autoaprendizaje No 1a 4/20/19, 4)33 PM

s_{\rm av} = 2.2 {\rm m/s }

Correct

Part C
Suppose that the ball moved in such a way that the graph segment after 2.0 {\rm s} was -3.0 {\rm{ m/s}} instead of +3.0
{\rm{ m/s}}. Find the ball's average velocity during the first 2.6 {\rm s} in this case.
Express your answer using two significant figures.

ANSWER:

v_{\rm av} = 0.85 {\rm m/s }

Answer Requested

Part D
Suppose that the ball moved in such a way that the graph segment after 2.0 {\rm s} was -3.0 {\rm{ m/s}} instead of +3.0
{\rm{ m/s}}. Find the ball's average speed during the first 2.6 {\rm s} in this case.
Express your answer using two significant figures.

ANSWER:

s_{\rm av} = 2.2 {\rm m/s }

Correct

Exercise 2.14

A race car starts from rest and travels east along a straight and level track. For the first 5.0 {\rm s} of the car's motion, the
eastward component of the car's velocity is given by \upsilon_x (t) = (0.870 \;\rm m/s^3) t^2.

Part A
What is the acceleration of the car when \texttip{\upsilon _x}{v} = 15.9 {\rm m/s} ?
Express your answer with the appropriate units.
ANSWER:

https://session.masteringphysics.com/myct/assignmentPrintView?assignmentID=7287775 Page 90 of 100


Taller de Autoaprendizaje No 1a 4/20/19, 4)33 PM

a_x = 7.44 \large{{\rm \frac{m}{s^{2}}}}

Correct

Exercise 2.23

The human body can survive a negative acceleration trauma incident (sudden stop) if the magnitude of the acceleration is
less than 250\;{\rm m}/{\rm s}^{2}.

Part A
If you are in an automobile accident with an initial speed of 113 {\rm km/h} and are stopped by an airbag that inflates
from the dashboard, over what distance must the airbag stop you for you to survive the crash?
ANSWER:

x = 1.97 {\rm m}

Correct

Exercise 2.44

A hot-air balloonist, rising vertically with a constant velocity of magnitude \texttip{v}{v} = 5.00 {\rm m/s} , releases a sandbag
at an instant when the balloon is a height \texttip{h}{h} = 40.0 {\rm m} above the ground . After it is released, the sandbag is
in free fall. For the questions that follow, take the origin of the
coordinate system used for measuring displacements to be at the
ground, and upward displacements to be positive.

Part A

https://session.masteringphysics.com/myct/assignmentPrintView?assignmentID=7287775 Page 91 of 100


Taller de Autoaprendizaje No 1a 4/20/19, 4)33 PM

Compute the position of the sandbag at a time 0.240 {\rm s} after its release.
ANSWER:

y = 40.9 {\rm m}

Correct

Part B
Compute the velocity of the sandbag at a time 0.240 {\rm s} after its release.
ANSWER:

v = 2.65 {\rm m/s}

Answer Requested

Part C
Compute the position of the sandbag at a time 1.50 {\rm s} after its release.
ANSWER:

y = 36.5 {\rm m}

Correct

Part D
Compute the velocity of the sandbag at a time 1.50 {\rm s} after its release.
ANSWER:

v = -9.70 {\rm m/s}

Correct

Part E
How many seconds after its release will the bag strike the ground?
ANSWER:

https://session.masteringphysics.com/myct/assignmentPrintView?assignmentID=7287775 Page 92 of 100


Taller de Autoaprendizaje No 1a 4/20/19, 4)33 PM

t = 3.41 {\rm s}

Correct

Part F
With what magnitude of velocity does it strike?
ANSWER:

v = 28.4 {\rm m/s}

Correct

Part G
What is the greatest height above the ground that the sandbag reaches?

ANSWER:

y = 41.3 {\rm m}

Correct

Exercise 2.52

The acceleration of a bus is given by a_{x}(t)= \alpha t, where \texttip{\alpha}{alpha} = 1.14 {\rm m/s^3} is a constant.

Part A
If the bus's velocity at time \texttip{t_1}{t_1} = 1.15 {\rm s} is 5.03 {\rm m/s} , what is its velocity at time \texttip{t_2}{t_2} =
2.11 {\rm s} ?
ANSWER:

v = 6.81 {\rm m/s}

Correct

Part B

https://session.masteringphysics.com/myct/assignmentPrintView?assignmentID=7287775 Page 93 of 100


Taller de Autoaprendizaje No 1a 4/20/19, 4)33 PM

If the bus's position at time \texttip{t_1}{t_1} = 1.15 {\rm s} is 5.94 {\rm m} , what is its position at time \texttip{t_2}{t_2} =
2.11 {\rm s} ?

ANSWER:

x = 11.5 {\rm m}

Answer Requested

PhET Tutorial: Moving Man

Learning Goal:
To understand the relationships between position, velocity, and acceleration.

NOTE: These activities use Java, and are therefore not screen-reader accessible and may not work on a mobile device. If
the browser you're using no longer supports Java, try a different browser and download the Java plugin for this content.

For this tutorial, use the PhET simulation The Moving Man. This simulation allows you to drag a person back and forth and
look at the resulting position, velocity, and acceleration. You can also enter a position as a function of time mathematically
and look at the resulting motion.

Start the simulation. When you click the simulation link, you may be asked whether to run, open, or save the file. Choose to
run or open it.

Under the Charts tab you can click and drag the person left and right, or enter a numeric value in the boxes on the left panel
to see plots for the person’s position, velocity, and acceleration as a function of time. Click the Play button to start a
simulation and the Pause button to stop a simulation. You can also watch a playback by selecting the Playback radio button
instead of the default Record radio button. You can click Clear to remove the current plot while maintaining your settings for
position, velocity and acceleration or click Reset All to start over. In the Playback mode, the grey bar can be dragged over
the plot to any value in time, and the digital readouts will show the corresponding values of the position, velocity, and
acceleration.

Under the Special Features menu, the Expression Evaluator option produces a second window in which you can

https://session.masteringphysics.com/myct/assignmentPrintView?assignmentID=7287775 Page 94 of 100


Taller de Autoaprendizaje No 1a 4/20/19, 4)33 PM

mathematically type in any function for the position as a function of time, \texttip{x\left(t\right)}{x(t)}. After typing in a function,
click the Play button to start the simulation.

To zoom in vertically, click any of the three + buttons to the top right of each plot. To zoom in horizontally, click the + button to
the bottom right of the acceleration plot.
Feel free to play around with the simulation. When you are done, click Reset All on the Charts tab before beginning Part A.

Part A
First, you will focus on the relationship between velocity and position. Recall that velocity is the rate of change of position
(v_x = dx/dt). This means that the velocity is equal to the slope of the Position vs. Time graph.

Move the person to the position x= -6\, {\rm m} or enter –6.00 in the position box. If you dragged the person to position,
click the Pause button and then the Clear button. Next, drag the person to the right to roughly x = 6\, {\rm m} and
reverse his direction, returning him to the original position, at x = -6\, {\rm m}. Move the person relatively quickly, about a
few seconds for the round trip. Your plots should look something like those shown below.

Look at the Position vs. Time and Velocity vs. Time plots. What is the person's velocity when his position is at its
maximum value (around 6 \rm m )?
ANSWER:

zero.
The person's velocity is negative.

positive.

https://session.masteringphysics.com/myct/assignmentPrintView?assignmentID=7287775 Page 95 of 100


Taller de Autoaprendizaje No 1a 4/20/19, 4)33 PM

Correct
When the person’s position is a maximum, the slope of the position with respect to time is zero, so dx/dt=0.
However, due to the person’s acceleration, the velocity does not remain zero; he eventually moves to the left.

Part B
Acceleration is the rate of change of the velocity, a_x=dv_x/dt, so it is the slope of the Velocity vs. Time graph.

Because it is difficult to drag the person in a consistent and reproducible way, use the Expression Evaluator under the
Special Features menu for this question.

Click Reset All and type in the function x(t) = 8*t-2*t*t in the Expression Evaluator. Click the Play button and let the
simulation run roughly 5 simulation seconds before pressing the Pause button. Use the zoom buttons to adjust the plots
so they fit in the screen. You should see a plot similar to what you got in the previous question, but much smoother.

Look at the Position vs. Time, Velocity vs. Time, and Acceleration vs. Time plots.

Hint 1. How to approach the problem

In Playback mode, use the grey vertical bar. Slide the bar until the value x = 8\, {\rm m} is displayed in the
position box on the left panel. What are the values of velocity and acceleration when x = 8\, {\rm m}?

ANSWER:

both the velocity and the acceleration are zero.

When the person is 8 \rm m to the right of the origin, the velocity is zero but the acceleration is negative.

both the velocity and the acceleration are nonzero.

the velocity is zero but the acceleration is positive.

Correct
At x=8\, {\rm m}, the person turns to go back in the opposite direction. His velocity is zero, but his acceleration
is negative since the velocity is decreasing with time. This is similar to throwing a ball straight up into the air; at
its highest point, the velocity is zero but the acceleration is still directed downward.

Part C
Keep the function x(t) = 8*t-2*t*t in the Expression Evaluator. What is the value of the person’s acceleration \texttip{a}{a}
at t=2\, {\rm s}?

Hint 1. How to approach the problem

https://session.masteringphysics.com/myct/assignmentPrintView?assignmentID=7287775 Page 96 of 100


Taller de Autoaprendizaje No 1a 4/20/19, 4)33 PM

Use the grey vertical bar. Slide the bar until it coincides with t=2\, {\rm s} on the horizontal axis. What is the value
displayed in the acceleration box on the left panel?

ANSWER:

-2\, {\rm m/s^2}

0
\texttip{a_{\mit x}}{a_x} =
-4\, {\rm m/s^2}

4\, {\rm m/s^2}

Correct
This is an example of one-dimensional motion with constant acceleration. The position of an object undergoing
this type of motion obeys the kinematic equation x(t) = x_0 + v_{x,0}t + 1/2\,a_x t^2. In this case, the initial
velocity is v_{x,0}=8\, {\rm m/s} and the acceleration is a_x=-4\, {\rm m/s^2} (since 1/2\,a_x =-2\, {\rm m/s^2}).

Part D
In the previous question, the person had an initial velocity of 8\, {\rm m/s} and a constant acceleration of -4\, {\rm m/s^2}.
How would the maximum distance he travels to the right of the origin change if instead his initial velocity were doubled
(v_{x,0}=16\, {\rm m/s})?

Hint 1. How to approach the problem

Go to the Introduction tab to run the simulation using the new initial velocity v_{x,0}=16\, {\rm m/s} and the same
acceleration of -4\, {\rm m/s^2}, and read the value for position when the velocity equals zero. Remember to
remove the walls from the simulation by clicking on the red close button on the walls. In Playback mode the
simulation can be run slowly and paused when the velocity is zero.

Or, mathematically, determine how long it takes for the person to stop and use this value of time in the equation
for x(t).

ANSWER:

The maximum distance would double.

The maximum distance would not change.

The maximum distance would increase by a factor of four.

https://session.masteringphysics.com/myct/assignmentPrintView?assignmentID=7287775 Page 97 of 100


Taller de Autoaprendizaje No 1a 4/20/19, 4)33 PM

Correct
Because it takes twice as much time to momentarily stop, and because his average velocity will be twice as
fast, the distance he travels will be four times greater. Using the kinematic equation, x(4\, {\rm s}) = (16\, {\rm
m/s})\times (4\, {\rm s})-(1/2)\times (4\, {\rm m/s^2})\times (4\, {\rm s})^2=32\, {\rm m}.

Part E
Now, assume that the position is given by the equation x(t) = 4\, {\rm t^3}.

Enter this function in the Expression Evaluator as x(t) = 4*t*t*t. Run the simulation by clicking the Play button in the
Record mode for roughly three simulated seconds and then click the Pause button. Now take a look at the graphs. You
will have to zoom in horizontally (bottom right), so that your range covers two seconds.

Which of the following statements is true?


ANSWER:

The position is increasing at a constant rate.

The velocity is increasing at a constant rate.

The acceleration is increasing at a constant rate.

The acceleration is constant in time.

Correct
The graph showing Acceleration vs. Time is a straight line that is not horizontal.

Part F
What is the position of the person when t=1\, {\rm s}?
Express your answer numerically in meters to one significant figure.

Hint 1. How to approach the problem

You can solve this mathematically using the expression input for position as a function of time, x(t) = 4*t*t*t.

Alternatively, you can put the simulation in Playback mode and drag the grey box/bar to t=1\, {\rm s}, then read
the approximate value for position.

ANSWER:

https://session.masteringphysics.com/myct/assignmentPrintView?assignmentID=7287775 Page 98 of 100


Taller de Autoaprendizaje No 1a 4/20/19, 4)33 PM

4 \rm m

Correct
Notice that since the position is given by x=4\, {\rm t^3}, when the time is t=1\, {\rm s}, the position is x =
4(1)^3\, {\rm m} = 4\, {\rm m}.

Part G
What is the velocity of the person when t = 1\, {\rm s}?
Express your answer numerically in meters per second to two significant figures.

Hint 1. How to approach the problem


The velocity is the first derivative of position with respect to time, v_x =dx/dt. You can take the derivative of the
expression of x(t) and evaluate when t = 1\, {\rm s}.

Alternatively, you can put the simulation in Playback mode and drag the grey box/bar to t=1\, {\rm s}, then read
the approximate value for velocity.

ANSWER:

12 \rm m/s

Correct
Notice that since the position is given by x(t)=4t^3, the velocity, which is the first derivative of position with
respect to time, is given by v_x = dx/dt = 12 t^2. So when t=1\, {\rm s}, v=12\, {\rm m/s}.

Part H
What is the acceleration of the person when t=1\, {\rm s}?
Express your answer numerically in meters per second squared to two significant figures.

Hint 1. How to approach the problem

The acceleration is the first derivative of velocity with respect to time, a_x =dv_x/dt or the second derivative of
position with respect to time, a_x = d^2x/dt^2. You can take the derivative of the expression of v_x(t) and evaluate
when t = 1\, {\rm s}.

Alternatively, you can put the simulation in Playback mode and drag the grey box/bar to t=1\, {\rm s}, then read
the approximate value for acceleration.

https://session.masteringphysics.com/myct/assignmentPrintView?assignmentID=7287775 Page 99 of 100


Taller de Autoaprendizaje No 1a 4/20/19, 4)33 PM

ANSWER:

24 {\rm m/s^2}

Correct
Notice that since the position is given by x=4 t^3, the acceleration, which is the first derivative of velocity with
respect to time, is given by a_x = dv_x/dt = d(12 t^2)/dx = 24 t. So when t=1\, {\rm s}, a_x=24\, {\rm m/s^2}.
Notice also that the acceleration is proportional to time, which explains why it is increasing at a constant rate
(as discovered in part G).

PhET Interactive Simulations


University of Colorado
http://phet.colorado.edu
Score Summary:
Your score on this assignment is 92.5%.
You received
Typesetting 30.52
math: 65% out of a possible total of 33 points.

https://session.masteringphysics.com/myct/assignmentPrintView?assignmentID=7287775 Page 100 of 100

You might also like